Sie sind auf Seite 1von 187

1.

A 32-year-old man presents with a 4-month


history of back pain. The pain is worse in the
morning and after sitting watching TV. Plain
radiograph of the spine/pelvis shows evidence
of sacroiliitis. Each of the following clinical
features might be identified on clinical
examination of this patient EXCEPT?
A Keratoderma
. blenorhegica
B Rheumatoid nodule
.
C Onycholysis
.
D Uveitis
.
E. Urethritis.
Correct Answer: B
Comment
The long (> 1 hr) early morning and rest stiffness are
highly suggestive of an underlying inflammatory
condition. The pelvic radiograph confirms the
presence of sacroiliitis. Bilateral sacroiliitis are typical
for ankylosing spondylitis. The frequency of
asymmetric sacroiliitis may be higher in other
spondyloarthropathies, e.g., reactive arthritis,
Reiters syndrome, spondylitis associated with
psoriasis, or inflammatory bowel disease.
Reiter's syndrome is characterised by a triad of
arthritis, urethritis, and conjunctivitis. Reiter's

syndrome develops in the setting of postdysenteric


or postvenereal illness. The characteristic rashes
of keratoderma blennorrhagicum and circinate
balanitis may be present.
Psoriatic spondyloarthropathy is characterised by
psoriatic plaques. The skin involvement may be
subtle and should be searched for carefully. The cleft
of the buttock, scalp hairline, and penis are site often
involved but may be missed easily if thorough
examination of these areas was not actively
conducted. Psoriatic nail changes include
onycholysis, yellow nails and nail pitting.
Inflammatory bowel disease consists of ulcerative
colitis and Crohn's disease.
A diagnosis of ankylosing spondylitis may be made
when specific features of Reiter's syndrome,
psoriasis, or inflammatory bowel disease are absent.
Sacroiliitis is also encountered in tuberculosis,
sarcoidosis and brucellosis.
Rheumatoid arthritis is not associated with lumber or
sacroiliac joint disease. However cervical spondylitis
and atlanto-axial subluxation is not an uncommon
feature of RA.
2. A 73-year-old man presents with a 24-hour
history of a painful swollen left knee. He has
had minor pain and stiffness in both knees for
many years and now feels generally unwell. His

temperature is 37.4C. What is the most likely


diagnosis?
A Gout
.
B Pseudogout
.
C Septic arthritis
.
D Reactive arthritis
.
E. Rheumatoid
arthritis.
Correct Answer: B
Comment
It would be important to exclude septic arthritis by
microscopy and culture of fluid aspirated from the
knee, but this is not the commonest cause of an
acute monoarthritis.
The differential diagnosis is influenced by age.
Crystal arthritis is the most common cause in elderly
people, whereas reactive arthritis tends to occur in
sexually active young adults.
Calcium pyrophosphate deposition disease,
often known as pseudogout, can often present with
striking fever and systemic illness. Treatment is with
nonsteroidal anti-inflammatory drugs (NSAIDs) or
intra-articular steroid injection, once sepsis has been
excluded.

3. (1) A 63-year-old man is admitted with


crushing central chest pain. He has previously
been reasonably well but has had troublesome
asthma for many years and has recently been
found to have hypertension. An ECG shows an
acute anterior myocardial infarction, which is
treated promptly with thrombolysis.
Two days later he is noted to have marked
livedo reticularis over his legs. Investigations
show his creatinine has risen from 115 umol/l
on admission to 280 umol/l. His blood count is
normal apart from an eosinophilia at 3x10*9/l.
His CRP is elevated at 40mg/l. Dipstick testing
of his urine is positive for blood and protein.
Which two of the following diagnoses seem
most likely?
A Goodpasture's syndrome
.
B Subacute bacterial
. endocarditis
C Giant cell arteritis
.
D Cholesterol emboli
. syndrome
E. Takayasu's disease

F. Churg Strauss syndrome


G Behcet's disease
.
H Aortic dissection
.
I. Systemic Lupus
Erythematosus
J. Rheumatoid vasculitis.
Correct Answer: DF
Comment
The combination of asthma, multisystem disease and
eosinophilia will suggest Churg Strauss vasculitis
to most rheumatologists/immunologists. However,
asthma is common and the occurrence of livedo
changes in the skin and renal impairment shortly
after vascular intervention or
anticoagulant/thrombolytic treatment is highly
suggestive of cholesterol emboli. The cholesterol
emboli syndrome can be accompanied by
immunological changes in the blood such as
eosinophilia, low complement concentrations and low
titre autoantibodies. The diagnosis is confirmed
histologically, with cholesterol clefts being seen in
affected vessels.

4. A 78-year-old man presents with a 2-day


history of severe headache localising to his
right temple. He also feels tired and lethargic.
He denies any eye problems and pain on
chewing. The biopsy of his right temporal
artery is shown (see image). Which of the
following statements fit best?
A This is a normal temporal artery biopsy.
.
B This man should be treated with 60 mg
. prednisolone.
C There is no need to consider bone
. protection.
D 50% of patients with this condition develop
. permanent visual loss.
E. An ESR of less than 40mm/hr excludes Giant
Cell Arteritis (GCA).
Correct Answer: B
Comment

This biopsy shows thickening and lymphocytic


infiltration highly suggestive of GCA. He should
indeed be treated with high dose steroid initially. It is
essential to consider bone protection as he is likely to
be on steroids for up to 2 years. (He may also need
gastric protection.) 15-20% of patients develop
permanent visual loss. The ESR is only elevated in
80% of cases, so if the history is good, the diagnosis
should still be considered.
5. A 35-year-old woman attends clinic
complaining of widespread pain and fatigue. A
history, examination and blood tests are
performed. Which two of the following would
be consistent with a diagnosis of fibromyalgia?
A Swollen joints
.
B Raised creatine kinase (CK)
.
C Raised thyroid-stimulating hormone
. (TSH)
D Raised elevated sedimentation rate
. (ESR)
E. Irritable bowel syndrome
F. Unrefreshing sleep
G Anaemia
.
H Impaired renal function

.
I. Proximal myopathy
J. Photosensitive rash.
Correct Answer: EF
Comment
Fibromyalgia is a diagnosis of exclusion. All blood
tests should be normal. Depression, irritable bowel
syndrome, unexplained headache and unexplained
chest pain are common in these patients.
6. A 26-year-old man with recurrent chest
infections is diagnosed as having common
variable immunodeficiency (CVID). From the
following, select the two most common
infectious agents responsible for disease in
this condition.
A S pneumoniae
.
B H influenzae
.
C E coli
.
D M tuberculosis
.
E. Enteroviruses
F. P Carinii

G Aspergillosis
.
H Nocardia
.
I. T gondii
J. L
monocytogenes
.
Correct Answer: AB
Comment
Ninety-five per cent of patients with CVID present
with recurrent sinopulmonary infections,
occurring as a result of encapsulated bacterial
infection, most of which are due to S pneumoniae
and H influenzae.
CVID patients are also predisposed to chronic
enteroviral infection, but fortunately this
complication is rare.
7. A 55-year-old woman presents with a 3month history of a painful swollen knee.
Examination shows restricted, painful
movement and a moderate sized effusion.
Synovial fluid is aspirated for diagnostic
purposes. Which two of the following findings
in the synovial fluid would be most compatible
with a final diagnosis of osteoarthritis?

A Macroscopically clear fluid


.
B Needle shaped crystals on microspcopy
.
C Highly viscous fluid
.
D High numbers of neutrophils on
. microscopy
E. Blood stained fluid
F. Thin, watery fluid
G High numbers of macrophages on
. microscopy
H Rhomboid-shaped crystals on
. microscopy
I. Macroscopically turbid
J. Gram positive cocci on gram stain.
Correct Answer: AC
Comment
Fluid aspirated from an osteoarthritic joint is
like normal synovial fluid: typically clear, viscous
and virtually acellular. Inflammatory fluid is turbid
(reflecting a high cell count) and loses viscosity
because of release of matrix degrading enzymes in
the synovial fluid.

8. A 67-year-old woman is referred to a


specialist rheumatology clinic. She has poorly
controlled rheumatoid arthritis, and for the
last fifteen years has been treated with various
disease-modifying drugs including
sulphasalazine and methotrexate. Blocking the
biological function of which one of the
following molecules will provide significant
anti-inflammatory effect in rheumatoid
arthritis?
A Interleukin-10 (IL-10)
.
B Interleukin-4 (IL-4)
.
C Interleukin-1 (IL-1)
.
D Soluble TNF receptors
.
E. Transforming growth factor-beta (TGFbeta)
Correct Answer: C
Comment
Advances in research have identified specific cells
and cell products (primarily cytokines) that may play
distinct roles in RA activity. It has been well
documented that in RA, proinflammatory cytokines
mediate synovial proliferation and articular tissue
destruction. For these reasons, cytokines have
become a target for therapy. Cytokine-based

therapies include a) blocking of proinflammatory


cytokines (IL1, IL2, IL6, IL8, TNFa, G-CSF, GM-CSF) or,
b) augmentation of anti-inflammatory cytokines (IL4,
IL10, IL11, IL13, TGFb, soluble TNF receptors, soluble
IL2 receptors).
Two of the major cytokines thought to incite
inflammation in RA patients are TNF and IL-1.
This has resulted in the introduction of two inhibitors
of TNF, the soluble TNF receptor construct Etanercept
and the anti-TNF monoclonal antibody (mAb)
Infliximab. More recently anti-IL-1 Kineret has
been licensed for use in active RA. On the other
hand IL10, IL-4 and TGF-beta may be considered a
natural dampeners of the immune response and
can be used as anti-cytokines. Therefore
augmentation of theses cytokines, rather than
blocking them, hold promise for new biologic therapy
in RA.

9. A 75-year-old retired farmer has been seen


in the outpatient clinic. He has been generally
unwell for the last month with fever and

weight loss, examination reveals this rash (see


image). While in the clinic he starts to cough
up blood and becomes acutely breathless. He
is admitted and initial investigations reveal a
serum creatinine of 170 micromol/l, a positive
myeloperoxidase (MPO) ELISA, negative
proteinase 3 (PR3) ELISA, negative GBM ELISA,
a negative ANA and normal complement levels.
Red cell casts are seen on microscopy of his
urine. Which two of the following diagnoses
are most compatible with the clinical picture?
A Sjogrens syndrome
.
B Goodpastures disease
.
C Lupus
.
D Henoch schonlein
. purpura
E. Cryoglobulinaemic
vasculitis
F. Polyarteritis nodosa
G Anti-phospholipid
. syndrome
H Wegener's
. granulomatosis
I. Microscopic polyangiitis
J. Takayasus arteritis.

Correct Answer: HI
Comment
This patient has presented with the pulmonaryrenal syndrome, which is most commonly caused
by the small vessel vasculitides (Wegeners
granulomatosis and microscopic polyangiitis (MP))
although Goodpastures disease, lupus and
cryoglobulinaemic vasculitis should also be included
in the differential diagnosis. ANCA positivity in this
patient group has a positive predictive value >90%
for diagnosing small vessel vasculitis. Although
specificity to MPO is more commonly associated with
microscopic polyangiitis, 25% of patients with
Wegners will also have a positive MPO. It is therefore
not possible to differentiate these two diseases,
based on ANCA specificity, nor is it important as
immunosuppressive treatment is indicated
regardless. A diagnosis of lupus is highly unlikely with
a negative ANA and >90% of patients with
cryoglobulinaemic vasculitis will have a low C4 as a
result of classical pathway activation.
10. A 70-year-old woman presents with
inflammatory joint pain, fatigue and a dry
mouth. She is found to have marked
hypergammaglobulinaemia and a raised
erythrocyte sedimentation rate with a normal
C-reactive protein.
What is the most likely diagnosis?
A Rheumatoid arthritis

.
B Pyrophosphate
. arthropathy
C Primary Sjgren's
. syndrome
D Systemic lupus
. erythematosus
E. Fibromyalgia
Correct Answer: C
Comment
Sicca symptoms, a raised erythrocyte sedimentation
rate but normal C-reactive protein (CRP) and
hypergammaglobulinaemia are classic features of
primary Sjgren's syndrome. This syndrome tends
to begin in the fifth and sixth decades compared with
lupus, which typically begins between the second
and fourth decades. This patient could have
rheumatoid arthritis but this is unlikely since the CRP
is normal. Blood tests are usually normal in patients
with primary fibromyalgia.
11. An 80-year-old woman is admitted having
tripped over a mat. Physical examination is
apparently unremarkable. She is unable to
weight bear and the Orthopaedic team have
discharged her with a normal pelvic
radiograph. She has been transferred to a
medical ward as she is still not able to walk.
What investigation(s) would you consider that

would most likely provide a definitive


diagnosis?
A Repeat standard pelvic radiograph and
. check a chemical profile.
B Bone scan or Magnetic Resonance Imaging
. (MRI).
C Dexa (dual energy radiograph
. absorptiometry) scan.
D Computerised Tomogram (CT) scan.
.
E. Myeloma screen.
Correct Answer: B
Comment
Diagnosis of hip fractures may not always be evident
on a plain radiograph. An anteroposterior view
obtained with the hip internally rotated 15 to 20
degrees will provide an optimal image and may
reveal a fracture not evident on on the standard ap
view. If there is a high index of suspicion of a
femoral fracture technetium-99 m bone
scanning or magnetic resonance imaging is
appropiate and will show a femoral fracture not
evident on a plain radiograph.
12. A 27-year-old woman has a 6-year history
of systemic lupus erythematosus (SLE) treated
with azathioprine, hydroxychloroquine and
prednisolone. She presents with a 2-week
history of slowly worsening severe pain and

restriction of the right hip. Which two of the


following diagnoses seem most likely?
A Gout
.
B Flare of systemic lupus erythematosus
. (SLE)
C Secondary osteoarthritis
.
D Perthe's disease
.
E. Septic arthritis
F. Secondary fibromyalgia
G Irritable hip
.
H Osteoporotic fracture of femoral neck
.
I. Avascular necrosis of the femoral head
J. Slipped upper femoral epiphyisis.
Correct Answer: EI
Comment
The most likely diagnosis is avascular necrosis, the
risk factors for this being corticosteroids and SLE
itself (particularly in patients with cardiolipin
antibodies). Sepsis is less likely but possible and
must be excluded. The risk of sepsis is increased by
immunosuppression (which may also modify the
presentation of sepsis- less fever, more insidious

onset, bloods may be normal).


This patient is at increased risk of osteoporosis but a
hip fracture would be a very rare event at this age,
and would usually be associated with sudden onset
of pain rather than insidious onset.
Osteoarthritis due to synovitis is almost never seen
in SLE which produces a non-erosive arthritis.
13. A 19-year-old girl develops muscle
weakness, a heliotrope rash and Gottron's
papules. The creatine kinase (CK) is raised. The
muscle biopsy confirms an inflammatory
myositis. A diagnosis of dermatomyositis is
made. Which of the following symptoms are
inconsistent with this diagnosis?
A Difficulty gripping
.
B Difficulty climbing stairs
.
C Difficulty turning over in
. bed
D Difficulty hanging out
. washing
E. Difficulty writing
F. Difficulty swallowing
G Difficulty lifting head off
. pillow
H Difficulty breathing
.

I. Difficulty getting out of


chair
J. Aifficulty dressing.
Correct Answer: AE
Comment
The myopathy in dermatomyositis and polymyositis
is classically a proximal myopathy. Patients may also
have truncal weakness and problems with the
muscles involved with swallowing and breathing.
14. A 75-year-old woman presents to the
Emergency Department with a hot swollen left
knee and difficulty in weight-bearing. She has
mild heart failure and is on treatment.
Which one of the following tests will be most
useful in determining how best to manage her
condition?
A Aspiration, microscopy and culture of
. synovial fluid
B Plain radiograph of knee
.
C Full blood count
.
D C-reactive protein
.
E. Serum uric acid
Correct Answer: A

Comment
In a patient presenting with a monoarthritis, the most
important diagnosis to consider/exclude is a septic
arthritis. This is best diagnosed by microscopy and
culture of the synovial fluid. Radiography will be
helpful in determining if the patient has pre-existing
osteoarthritis or chondrocalcinosis. A raised white
cell count might suggest infection. A raised Creactive protein would indicate inflammation or
infection. This patient has mild heart failure and so
may be taking bendroflumethiazide or loop diuretics,
which may cause hyperuricaemia and gout and
hence a monoarthritis.

15. A 70-year-old diabetic had problems with


decreased mobility of his back. The problem
had progressively worsened over the last
seven years. His lumbar spine was remarkable
for loss of the normal lordosis and decreased
range of motion in all planes. He had no pain
on percussion over the spine or sacroiliac
joints. He was otherwise in good health, and
the erythrocyte sedimentation rate (ESR) was
normal. A radiograph of the lumbar spine is
shown. The most probable diagnosis is:
A Ankylosing spondylitis (AS)
.
B Alcaptonuria
.
C Osteoporosis
.
D Spondylolisthesis
.
E. Diffuse idiopathic skeletal hyperostosis
(DISH)
Correct Answer: E
Comment
Radiographs of the lumbar spine showed only
minimal degenerative disc disease, but large, coarse
osteophytes that bridged the lower vertebral bodies.
These findings are classic for DISH. Criteria for DISH
require that new bone formation bridge four
contiguous vertebral bodies in the absence of

significant degenerative disk disease, and in the


absence of inflammatory sacroiliac or facet changes.
There is increased prevalence of DISH in
diabetes mellitus and obesity.
The lack of sacroiliac tenderness in this man, with
longstanding symptoms, normal sedimentation rate
and advanced age of onset, rendered AS an unlikely
diagnosis.
Alcaptonuria is a hereditary disease in which
homogentisc acid oxidase activity is missing.
Homogenitisc acid accumulates and cause
pigmentation of cartilage and other connective
tissues (ochronosis). It usually starts in the second or
third decade. The vertebral bodies of the lumbar
spine show degeneration of the intervertebral discs,
with narrowing of the space and dense calcification
of remaining disc material. There is variable fusion of
vertebral bodies, but little osteophyte formation and
minimal calcification of intervertebral ligaments.
Osteoporosis is associated with reduced bone
density and wedge fractures of the vertebrae.
Spodylolisthesis is forward movement of one
vertebra in relation to an adjacent vertebra.
16. A 30-year-old woman presents with several
attacks of pain and swelling affecting the
metacarpophalangeal and proximal
interphalangeal joints over the past few
months, with morning stiffness and fatigue.
What are the two most likely diagnoses?

A Ankylosing spondylitis
.
B Pseudogout
.
C Systemic lupus erythematosus
. (SLE)
D Primary generalized
. osteoarthritis
E. Reactive arthritis
F. Gout
G Rheumatoid arthritis
.
H Psoriatic arthropathy
.
I. Systemic sclerosis
J. Rheumatic fever.
Correct Answer: CG
Comment
The most likely cause of these symptoms is
rheumatoid arthritis, but SLE requires careful
consideration.
Rheumatoid arthritis can present as an acute
monoarthritis, an acute polyarthritis, a subacute
insidious polyarthritis, or with a polymyalgic
presentation (particularly in the elderly, when it
needs to be differentiated from polymyalgia

rheumatica).
Remember that acute synovitis in early rheumatoid
arthritis is associated with very little fixed joint
deformity and no extra-articular features. This
contrasts with the deforming arthritis that is
characteristic of chronic disease.
17. A 30-year-old woman presents with a 3month history of recurrent episodes of
urticaria. There are no clues in the history to
suggest a possible trigger. During each episode
she is tired but otherwise well and clinical
examination is unremarkable. What is the most
likely diagnosis?
A C1 inhibitor deficiency
.
B Urticarial vasculitis
.
C Food allergy
.
D Systemic mastocytosis
.
E. Chronic idiopathic
urticaria.
Correct Answer: E
Comment
The cardinal presenting feature of C1 inhibitor
deficiency is angioedema.

Urticarial vasculitis is characterized by painful


wheals often lasting more than 24 hours and
accompanied by systemic symptoms.
In the absence of any clues in the history, this
patient's urticaria is unlikely to be due to food allergy.
Mastocytosis is very rare; the urticaria associated
with mastocytosis is invariably accompanied by
Darier's sign and GI involvement.
18. A 34-year-old woman presents to your
clinic complaining of cold hands, particularly in
the winter months. On examination she has
cold dusky hands, splinter aemorrhages and a
petechial rash. Investigations are as follows:
Hb 10.9 g/dL; Wbc 4.2 x 109/L; Platelets 407 x
109/L. Antinuclear antibodies present (1/160).
DNA antibodies not detected. Complement C3
0.79 (NR 0.75-1.25), C4 <0.04 (NR 0.14-0.6).
Which of the following investigations are not
indicated?
A Liver function tests
.
B Skin biopsy
.
C Cryoglobulins
.
D Urinanalysis
.

E. Hepatitis C antibodies
F. Extractable nuclear antibodies
(ENA)
G Endomysial antibodies
.
H Thyroid antibodies
.
I. Rheumatoid factor
J. Serum creatinine.
Correct Answer: GH
Comment
This womans symptoms are suggestive of
cryoglobulinaemia and the low C4 would be
compatible with complement consumption due to
mixed or polyclonal cryoglobulins. Cryglobulins are
proteins that precipitate at temperatures lower
than 37C. This results in sludging of the circulation
in the extremities, and sometimes vasculitis,
especially of skin and kidneys. The commonest
causes are hepatitis C and Sjrgrens syndrome.

19. A 13-year-old boy was hospitalised because


of a purpuric rash involving the legs (see
image) associated with abdominal pain and
fever (38C). The patient also complained of
arthralgia involving the knees and ankles.
Urinalysis showed proteinuria with microscopic
haematuria. A biopsy of the purpuric lesion
revealed leucocytoclastic vasculitis in the small
vessels. Which of the following statements is
true about this boy's illness?
A The urine abnormality and fever is most
. probably related to a recent urinary tract
infection
B In such cases blood cultures are often
. positive for Pseudomonas aerogenosa
C Renal biopsy typically shows mesangial IgA
. deposition

D Bilateral small kidneys are a constant


. feature on ultrasound of the kidneys
E. Untreated, up to 50% will develop chronic
renal failure.
Correct Answer: C
Comment
Henoch-Schonlein Purpura (HSP) is recognized as
a systemic small vessel vasculitis involving mainly
the blood vessels of the skin, GI tract, the kidneys
and the joints.
HSP affects mainly children between the ages of
three and 10 years. Males are affected more often
than females (1.5:1) and in about two thirds of
children an upper respiratory tract infection precedes
the onset of HSP by one to three weeks.
Kidney involvement in HSP is secondary to vasculitis
which is not associated with previous or concurrent
urinary tract infection. HSP nephritis becomes
clinically manifest in only 20-30%. It usually presents
as macroscopic haematuria and proteinuria lasting
from a matter of days to several weeks. The kidneys
are usually of normal size. Of those patients with
renal involvement, as many as 10% may develop
chronic renal failure and end-stage renal disease.
However, fewer than 1% of all patients with HSP
suffer this poor prognosis.

20. A 23-year-old woman is brought by


ambulance to the Emergency Department
having collapsed in a restaurant while eating a
curry. Her friends have given a history of
previous allergic reactions to nuts. On
admission she is flushed, breathless and
wheezy, has a pulse rate of 140 bpm and a BP
of 84/40 mmHg.
What is the most appropriate first line of
treatment?
A High-flow oxygen, intravenous
. hydrocortisone and chlorpheniramine
B High-flow oxygen and intravenous
. epinephrine 500 g
C 24% oxygen and intramuscular epinephrine
. 500 g
D 1 litre intravenous saline and intravenous
. chlorpheniramine
E. High-flow oxygen and intramuscular
epinephrine 500 g
Correct Answer: E
Comment
Intramuscular epinephrine is the key treatment
in anaphylaxis. Intravenous epinephrine may
occasionally be used with extreme caution in
patients with cardiovascular collapse, but it is more
likely to cause ventricular arrhythmias when
administered by this route. Antihistamines and

corticosteroids (without epinephrine) are


inadequate immediate treatments for
anaphylaxis, but may be administered in milder
allergic reactions and also to prevent late
deterioration in the event of anaphylaxis.
21. A 25-year-old African man with known HIV
infection presents with fever, cervical
lymphadenopathy, splenomegaly and renal
impairment. Significant findings on laboratory
testing include a strongly positive antineutrophil cytoplasmic antibody (ANCA) of
proteinase-3 specificity (PR3-ANCA). The
positive predictive value of his PR3-ANCA for a
diagnosis of small vessel vasculitis is
approximately:
A <10%
.
B 10-20%
.
C 20-40%
.
D 40-60%
.
E. >60%.
Correct Answer: A
Comment
False-positive ANCA antibodies are a well-recognised
pitfall in this situation. A variety of infections are

known to be associated with ANCA including bacterial


endocarditis and a range of tropical infections
including Amoebiasis and Malaria. It underlines the
need to obtain histological evidence of small vessel
vasculitis before proceeding to immunosuppressive
therapy.
23. A 20-year-old woman presented with
sudden onset of swelling of the lips and
tongue. She also had abdominal pain and
vomiting. Her mother confirmed that her
daughter had similar attacks over the years
and even as a child. A brother and older sister
have the same disorder. Which of the following
statements about this disease is accurate?
A It has sex-linked inheritance.
.
B Animal allergen is often identified in the
. house
C Serum C4 levels are often low.
.
D Antinuclear antibodies (ANA) is often
. positive.
E. Raised IgE helps differentiate it from other
immune disorders.
Correct Answer: C
Comment
Hereditary angioneurotic oedema is an
autosomal dominantly inherited condition caused by

a deficiency of C1 esterase inhibitor. This results in


intermittent episodes of spontaneous complement
activation. Clinically the patient suffers oedema of
the skin and mucosal surfaces. Fatalities may occur if
the airway is compromised. C4 levels are typically
low during an attack; they may be normal in
between attacks.
Acquired angioedema (AAE) is angioedema
associated with allergic reactions, which is often
associated with urticaria. Approximately 94% of
cases of angioedema are drug-induced. Most are
patients taking angiotensin-converting enzyme (ACE)
inhibitors. Insect stings, and foods are other
predisposing factors
24. (2) A 29-year-old man presents with
palpable purpura and large joint arthritis. He is
systemically well. Dipstick testing shows blood
and protein in the urine. Blood tests
reveal that his serum creatinine is elevated at
200 micromol/l, and liver blood tests show an
ALT of 120 iu/l (normal <40). Immunological
investigations show a positive test for
rheumatoid factor at 1/256 (normal <1/32),
negative ANA and low C4 at 0.02g/l (normal
0.2-0.6). Which two of the following diagnostic
tests are most likely to lead to a definitive
diagnosis?
A Antineutrophil cytoplasmic antibody
.
B Antiglomerular basement membrane
. antibody

C Serum immunoglobulins and


. electrophoresis
D Antistreptolysin O
.
E. Echocardiogram
F. Cryoglobulins
G Blood cultures
.
H Ultrasound of the kidneys and renal
. tract
I. Hepatitis C serology
J. Hepatitis B serology.
Correct Answer: FI
Comment
The clinical picture is highly suggestive of
glomerulonephritis associated with systemic
vasculitis or infection. All the listed investigations
may be indicated in this clinical situation. However,
combination of high titre rheumatoid factor,
negative ANA, very low C4 and abnormal LFTs
raises very strong suspicions of a mixed
cryoglobulinaemic vasculitis associated with
hepatitis C infection. Further questioning revealed
a brief history of intravenous drug use 6 years
previously and investigation confirmed the clinical
diagnosis. Subsequent renal biopsy showed
aggressive glomerulonephritis with changes
suggestive of cryoglobulinaemia.

25. A 56-year-old man presents with a 2-month


history of progressive mid-thoracic spinal pain
that disturbs his sleep. A CXR is normal. His
erythrocyte sedimentation rate is 60 mm/hour
(normal <10 mm).
Which of the following investigations is most
likely to clarify the cause of his pain?
A Isotope bone scan
.
B MRI scan of the thoracic spine
.
C CT scan of the thoracic spine
.
D Myeloma screen
.
E. Plain radiograph of the thoracic
spine
Correct Answer: B
Comment
The clinical picture suggests malignancy or
chronic infection, although sometimes
osteoarthritis may produce severe pain.
Osteoporotic crush fractures usually cause
pain of sudden onset. An isotope bone scan may
reveal the site of the painful lesion but is unlikely to
reveal the cause, and may be normal in myeloma.

Plain radiographs and CT scans will show


evidence of advanced disease but may be normal in
early malignancy and infection. MRI scanning is
the most sensitive technique, and will also give
information about associated soft tissues.
26. A 19-year-old woman collapses in a
restaurant, shortly after beginning her meal.
Which of the following statements is true?
A Anaphylaxis is not the cause if neither
. stridor nor wheeze is audible.
B If pallor and hypotension are present:
. anaphylaxis is the likely diagnosis.
C If she has facial swelling and wheeze:
. anaphylaxis is likely even if there is no
hypotension.
D A history of chronic urticaria implies that
. she is allergic to something.
E. Scrombrotoxin poisoning is a possibility and
will not respond to intramuscular
epinephrine.
Correct Answer: C
Comment
Anaphylaxis is caused by histamine release in
response to contact with an allergen. In sensitized
people, specific IgE attached to mast cells binds to
ingested or inhaled allergen, causing the mast cell to
degranulate, releasing histamine. If sufficient
histamine is released, anaphylaxis occurs. The
symptoms of anaphylaxis are facial and laryngeal

oedema, wheeze and hypotension. Uriticaria and


flushing may also occur. Not all features are present
in every case. Acute urticaria in this situation would
be suggestive of anaphylaxis but chronic urticaria is
usually non-allergic in nature.
Pallor and hypotension are features of a simple
faint. Patients have a slow weak pulse. In situation B
this is the most likely diagnosis unless additional
features of anaphylaxis are present.
Scrombrotoxin poisoning occurs when fishes such
as mackerel or tuna are spoiled, when they may
produce large amounts of histamine. Unlike
anaphylaxis, where symptoms are usually fairly
immediate, signs of scrombrotoxin poisoning occur at
least an hour after ingestion of the suspect food.
Symptoms would be expected to respond to
epinephrine.
27. (2) A 34-year-old man presents with severe
low back pain, which has forced him to stop
work as a bus driver. He has had back pain on
and off for many years, on occasion with rightsided sciatica. The pain used to be helped by
rest, but is now present more or less all the
time and is stopping him from sleeping
properly. The most likely diagnosis is:
A mechanical back
. pain
B ankylosing
. spondylitis

C myeloma
.
D osteoporosis
.
E. osteoarthritis.
Correct Answer: A
Comment
The history is typical of a patient with mechanical
back pain, best treated by encouraging
mobilization, simple analgaesia, a graded
rehabilitative exercise programme and treatment of
depression (if present).
Red flag symptoms, requiring urgent
investigation to exclude sinister pathology, include:

age >55 or <18 years

progressive pain

night pain

systemic symptoms

progressive neurological deficit

past history of malignancy or


immunosuppression

recent trauma.
28. A 25-year-old man presents with a 3-week
history of haemoptysis and testicular pain. The
only significant findings on examination are
testicular tenderness and left-sided
episcleritis. The results of initial investigations
are as follows: serum CRP 124 g/l (nr <8), urine

sediment: red cell casts, serum creatinine 78


micromol/L, serum C3 124 mg/dl (nr 65-185),
serum C4 32 mg/dl (nr 15-50), antinuclear
antibody negative, chest radiograph: bilateral
nodular shadowing. Which of the following
diagnoses is most likely to account for his
clinical presentation?
A Systemic lupus erythematosus (SLE)
.
B Small cell lung carcinoma
.
C Mixed cryoglobulinaemia
.
D Small vessel vasculitis of the Wegener's
. microscopic polyangiitis spectrum
E. Giant cell arteritis.
Correct Answer: D
Comment
This patient has an inflammatory multi-system
disorder of relatively short duration affecting his
lungs, kidneys, eyes and testes. The constellation of
nodular lung shadows, glomerulonephritis (as
evidenced by red cell casts in the urine), episcleritis,
testicular tenderness and an elevated CRP is highly
suggestive of an ANCA+ necrotising small vessel
vasculitis. Systemic immune-complex diseases such
as SLE and mixed cryoglobulinaemia are excluded by
the negative ANA and normal complement profile,
respectively.

This patient needs assessment of his ANCA status


and tissue biopsy-either kidneys or lungs, preferably
kidneys.
29. A 40-year-old woman suddenly develops
severe Raynauds that is troublesome in the
summer as well as the winter. She starts to
develop swelling of the fingers and feels tired.
She also develops reflux oesophagitis and has
difficulty swallowing. She has noticed that she
has become more breathless. On examination,
she has skin thickening affecting her hands,
face and trunk. Which of the following is most
likely to be positive?
A Ds DNA antibody
.
B Ro antibody
.
C Anticentromere
. antibody
D Jo-1 antibody
.
E. Scl-70 antibody.
Correct Answer: E
Comment
This patient is developing diffuse systemic
sclerosis. Anticentromere pattern is associated with
limited systemic sclerosis. Ro antibody is common in
patients with primary Sjogren's syndrome and

systemic lupus erythematosus (SLE). Jo-1 may be


positive in patients with polymyositis.
30. A 68-year old man presented with sudden
severe pain and swelling in the left knee.
Synovial fluid analysis shows abundant calcium
pyrophosphate dihydrate (CPPD) crystals.
Which of the following tests is NOT appropriate
for further assessment of this patient illness?
A Creatinine kinase
.
B Serum calcium
.
C Thyroid function
. test
D Serum ferritin
. level
E. Hb AIc.
Correct Answer: A
Comment
Pseudogout describes acute attacks of CPPD
crystal-induced synovitis which clinically resembles
urate gout. However, the majority of individuals
with CPPD crystal deposition never experience
such episodes.
Chondrocalcinosis is the term used to describe the
calcium-containing deposits that are found in
cartilage and which are usually visible on joint
radiographs. A variety of metabolic and endocrine

disorders are associated with CPPD crystal


deposition, which include:

diabetes mellitus,

hemochromatosis,

Wilson's disease,

hypothyroidism,

hyperparathyroidism,

hypomagnesemia,

hypophosphatasia.
31. A 30-year-old man has a swollen wrist, low
back pain with early morning stiffness lasting 2
hours and mouth ulcers. He does not have a
rash. What is the most likely diagnosis?
A Rheumatoid arthritis (RA)
.
B Psoriatic arthritis
.
C Ankylosing spondylitis
.
D Gout
.
E. systemic lupus erythematosus
(SLE)
Correct Answer: C
Comment
The history is suggestive of a seronegative
spondyloarthropathy.

RA is classically a peripheral, symmetrical


polyarthritis.
Gout can affect the wrist but is unusual in a
young person.
Patients with SLE can experience mouth ulcers but
SLE is uncommon in young males.

32. A 52-year-old accountant, with a 6-year


history of Raynauds phenomenon presents to
her G.P. with a 4-month history of worsening
dysphagia. Incubation of her serum on Hep 2
cells reveals the following immunofluorescent
pattern (see image).
A Primary Raynauds disease
.
B systemic lupus erythematosus
. (SLE)
C Limited systemic sclerosis

.
D Rheumatoid arthritis
.
E. Sjogren's disease.
Correct Answer: C
Comment
Raynauds phenomenon is common, occurring in
4-15% of the general population and in the majority
of cases is not associated with connective tissue
disease. However, the detection of an anticentromere nuclear staining pattern has a high
specificity for limited systemic sclerosis,
especially in the context of dysphagia which is a
feature of the disease.
33. An 18-year-old student with known asthma
and peanut allergy collapses following a meal
in the hospital canteen. Strenuous attempts at
resuscitation are tragically unsuccessful. It is
thought that the most likely cause of his
collapse was an anaphylactic reaction to
hidden' nuts contained in the meal.
Which of the following mast cell-derived
mediators is it most useful to measure to
confirm the clinical suspicion of anaphylaxis?
A Mast cell - derived growth
. factor
B Chymase
.

C Interleukin-6
.
D Tryptase
.
E. Transforming growth factor
Correct Answer: D
Comment
Amongst the array of mediators released by mast
cells during anaphylaxis, tryptase has proven to be
a reliable marker of mast cell degranulation on
account of its stability, relatively long half-life and
ease of measurement. Consequently, an elevated
tryptase in the correct clinical context is a useful
surrogate marker of an anaphylactic/anaphylactoid
reaction.
34. A 60-year-old woman develops acute,
severe low thoracic back pain. A radiograph
shows a vertebral crush fracture. Which two of
the following reduce an individual's risk for
developing osteoporosis?
A Early menarche
.
B Early menopause
.
C Smoking
.
D High alcohol intake

.
E. Prolonged treatment with
corticosteroids
F. Rheumatoid arthritis
G Crohn's disease
.
H Asthma
.
I. Frequent walks
J. Obesity.
Correct Answer: AI
Comment
The most likely cause of a vertebral crush factor in a
woman of this age is obviously osteoporosis, but
local vertebral pathology - in particular secondary
tumours or myeloma - would need to be considered.
It is most likely that the cause of osteoporosis would
be postmenopausal in this woman, but many medical
factors influence the likelihood of this condition.
The following tests should be considered in a
patient presenting with an osteoporotic
fracture (although not all need to be performed in
all cases): full blood count, renal / liver / bone
chemistry, immunoglobulins and serum / urine
electrophoresis (to exclude myeloma), thyroid
function, testosterone (in men), investigations for
Cushing's syndrome.

35. This patient was trying to stand up. On


examination he has periorbital oedema with a
faint purple hue. The serum creatinine kinase
(CK) levels were 10-times the upper limit of
normal. Which of the following manifestations
is least likely to be associated with this
disorder?
A blue and white color change in the fingers
. on exposure to cold
B difficulty in swallowing liquids
.
C double vision on looking to the sides

.
D radial pulse of 40 beats per minute
.
E. persistent cough, haemoptysis and weight
loss of 12 kg in the last three months
Correct Answer: C
Comment
This patient is not able to stand up from a chair
unaided. This indicates proximal muscle
weakness. The skin rash and the raised levels of
muscle enzyme point towards an inflammatory
myopathy, namely dermatomyosytis (DM).
DM is easily recognized and diagnosed by a
characteristic rash:

a heliotrope rash (blue-purple discoloration) on


the upper eyelids, with oedema

a flat red rash on the face and upper trunk.


Erythema of the knuckles accompanied by a
raised, violaceous scaly eruption (Gottron's
sign) is also characteristic, and may precede or
follow muscle weakness. Raynaud's phenomenon
with characteristic color change (pallor, cyanosis and
rubor) on exposure to cold is encountered in patient
with DM, more often when there is overlap with other
connective tissue disease such as scleroderma or
mixed connective tissue disease.
Inflammation of the skeletal muscles of the
oropharynx and upper oesophagus lead to
dysphagia especially for liquids in the initial stages
of the disease which progresses to difficulty in

swallowing solids as the disease progresses.


Electrocardiographic evidence of conduction
defects and arrhythmias occur frequently in
childhood and adult DM, although overt clinical
symptoms are uncommon.
There is an increased incidence of underlying
malignancy in adult DM, ranging from 5% to 15%.
Carcinoma of the bronchus, ovaries and breast
are the most common.
Ocular muscles remain normal, even in
advanced, untreated cases, and if these muscles are
affected, the diagnosis of inflammatory myopathy
should be in doubt.
36. A 34-year-old woman presents to your
clinic complaining of cold hands, particularly in
the winter months. On examination, she has
cold dusky hands and a petechial rash.
Investigations are as follows: Hb 10.9 g/dL;
Wbc 4.2 109/L; Platelets 407 x 109/L; urea and
electrolytes - normal; liver function tests
- normal; albumin 36 g/L; globulin 90 g/L;
protein electrophoresis - polyclonal increase in
gammaglobulins; antinuclear antibodies
present (1/160, speckled pattern); complement
- C3 0.79 (NR 0.75-1.25), C4 <0.04 (NR 0.140.6). Which of the following statements is true?
A Active systemic lupus erythematous (SLE) is
. unlikely if DNA antibodies are present.
B A blood sample sent to the lab on ice may
. show cryoglobulins.
C Sjorgrens syndrome is unlikely if
. rheumatoid factor is present.

D Sjorgrens syndrome is likely if Ro and La


. extractable nuclear antigens are present.
E. Hepatitis C is unlikely in this case.
Correct Answer: D
Comment
The symptoms and signs and low C4 are
suggestive of cryoglobulinaemia.
Sludging of proteins at reduced temperatures (for
example hands on a cold day) can cause ischaemia
and sometimes vasculitis, particularly of skin or
kidneys.
Cryoglobulinaemia is commonly associated with
hepatitis C or connective tissue disease, such as
Sjorgren's syndrome. The positive ANA and high
globulins suggest Sjorgren's but could also be
associated with chronic infection, such as hepatitis C.
Since they precipitate at low temperatures,
cryoglobulins should always be transported to the lab
at 37C. Failure to do this will result in a false
negative result as the cryos will precipitate and be
removed with the clot.
37. A 69-year-old man with a 10-year history of
rheumatoid arthritis, controlled with diclofenac
and sulphasalazine has reported recurrent
heart burn and dyspepsia. His doctor asked
him to stop the dicofenac sodium and replaces
it with celecoxib (one of the COX 2 inhibitors).
Which of the following is true about COX 2
selective agents?

A They eliminate the chance of non-steroidal


. anti-inflammatory drugs (NSAID)-associated
dyspepsia.
B They are now regarded as the primary
. treatment for rheumatoid arthritis.
C They are more effective than ordinary
. NSAIDs.
D Because of their potent antiplatelet effect
. the addition of low dose aspirin for
ischaemic heart disease prophylaxis is not
necessary.
E. They can cause acute renal failure when
used in patients with impaired renal
function.
Correct Answer: E
Comment
NSAIDs inhibit prostaglandin synthesis by inhibiting
the enzyme cyclooxygenase (COX). The existence of
two different forms of COX isoenzymes (COX-1 and
COX-2) has now been recognised. COX-1 has a
constitutive 'housekeeping' role and COX-2
upregulation is associated with inflammation. The
COX-2-selective NSAIDs are not more effective than
ordinary NSAIDs, they are not the primary treatment
for rheumatoid arthritis, and they do not eliminate
the chance of NSAID-associated dyspepsia. Unlike
aspirin and most traditional NSAIDs, they have no
antiplatelet effect, so they are not cardioprotective.
In clinical trials COX-2 selective NSAIDS reduced the

risk of developing significant upper gastrointestinal


complications in the first six months. However, the
risk of GI bleeding from nonselective NSAIDs is low an annual incidence of 1% to 3% - and the COX-2selective NSAIDs are costly. Therefore, it may be best
to reserve these drugs for patients at high-risk
patients to include those over 65 years, those
already using medications known to increase the
likelihood of upper gastrointestinal adverse events,
those with serious co-morbidity, and those requiring
prolonged use of maximum recommended doses of
standard NSAIDs.
All NSAIDs, including the COX-2 selective agents, can
cause renal toxicity, such as acute renal failure,
nephrotic syndrome, and acute interstitial nephritis.
Acute renal failure is a concern particularly in
patients with pre-existing renal dysfunction. NSAIDs
should be avoided also in patients with cirrhosis and
ascites or congestive heart failure. Periodic
monitoring of renal function is recommended,
particularly in elderly patients.
38. A 36-year old woman is referred with a 1year history of muscle pain, tiredness and
sleep disturbance. She denies fever, weight
loss and arthralgia. Examination reveals
tenderness over her occiput, trapezius and
lumbar area. Her blood results show a normal
ESR, CRP, FBC, a weakly positive ANA 1:80 and
normal complement. Which is the most likely
diagnosis?
A Polymyositis
.

B System lupus erythematous


. (SLE)
C Sjogrens syndrome
.
D Polymyalgia rheumatica
.
E. Fibromyalgia
Correct Answer: E
Comment
The lack of constitutional symptoms, normal
inflammatory markers and normal examination, apart
from evidence of tender points, make an
inflammatory rheumatological disease unlikely. The
presence of tender points, history of muscle pain and
sleep disturbance are suggestive of fibromyalgia
a non-inflammatory pain disorder.
39. A 25-year-old woman with a history of 3second trimester fetal losses is planning a
fourth pregnancy. She has evidence of the
primary anti-phospholipid syndrome (strongly
positive cardiolipin antibody, positive lupus
anticoagulant but no evidence of lupus). Which
of the following treatment regimens offer her
the best chance of having a successful
pregnancy?
A Steroids alone
.
B Steroids combined with low-dose aspirin

.
C Low-dose aspirin alone
.
D Low-dose aspirin combined with low
. molecular weight heparin
E. Intravenous immunoglobulin.
Correct Answer: D
Comment
Of the above treatment options, low-dose aspirin
combined with low molecular weight heparin offers
her the best chance of having a successful
pregnancy.
40. You are called to A&E to review a 28-yearold woman with pleuritic chest pain. Which of
the following statements concerning this
patient are true?
A Low white count is consistent with systemic
. lupus erythematosus (SLE)
B Low white count is not consistent with
. tuberculosis.
C The presence of anticardiolipin antibodies
. IgG 17 GPLU/ml (NR <14), IgM 21 MPLU/ml
(NR<10) suggests that she may have a
pulmonary embolus.
D The presence of antinuclear antibodies at a
. titre of 1/80 suggests that she may have a
connective tissue disease.

E. The presence of antinuclear cytoplasmic


antibodies (cANCA) at a titre of 1 in 20
suggests that Wegener's granulomatosis is
likely.
Correct Answer: A
Comment
Active SLE and tuberculosis are both
associated with leukopenia. Low titre antinuclear
antibodies (in the absence of DNA or ENA),
antineutrophil cytoplasmic antibodies (ANCA) and
anticardiolipin antibodies are non-specific and are
commonly found in the presence of infection.
41. A 66-year-old woman has a 15-year history
of deforming rheumatoid arthritis (RA). She is
maintained on D-Penicillamine. Two weeks ago
she noticed increased difficulty in climbing
stairs and three days before admission she was
unable to comb her hair or feed herself.
Neurological assessment is difficult
but reveals generalised grade 3/5 weakness,
and the Babiniski sign is positive bilaterally.
Which one of the following tests is most likely
to reveal the diagnosis?
A Plain radiograph of the cervical spine
.
B Electromyography (EMG)
.
C Nerve conduction study (NCS)
.

D Isotope bone scan


.
E. Magnetic resonance imaging (MRI) of the
cervical spine
Correct Answer: E
Comment
The recent limb weakness and the presence of
pyramidal signs in the legs in patients with RA are
highly suggestive of spinal cord lesion at the
cervical or thoracic region.
RA primarily affects the cervical spine; affliction of
the thoracic or lumbar spine is rare. The anatomical
abnormalities occur as a consequence of the
destruction of synovial joints, ligaments, and bone.
Atlantoaxial subluxation (AAS) is the most
common. Patients may experience weakness,
decreased endurance, gait difficulty, paresthesias of
the hands, and loss of fine dexterity. Multiple
neurological signs may be elicited on physical
examination, including diffuse hyperreflexia, lower
extremity spasticity, a spastic gait, and Babinski's
sign.
Although plain radiography of the cervical spine is
regarded as the initial imaging assessment tool for
neck pain in patient with RA, those with symptoms or
signs of cord compression should undergo immediate
MRI and be sent for surgical consultation. MRI is
considered the most sensitive imaging modality for
diagnosing spinal cord compression, evaluating its
extent, and assessing soft tissues (panus) as well as
bone destruction.

D-Penicillamine causes various neuromuscular


disorders including myathenia gravis.
There is no clinical evidence for primary muscle
disease or peripheral neuropathy in this patien,
hence EMG or NCS are not necessary.
Isotope bone scan may identify increased activity in
the cervical region, but this is not specific and the
investigation has little role in investigation of
suspected spinal cord compression.
42. A 70-year-old woman with a history of
blood transfusion in the early 1980s presents
with a 10-month history of malaise and is
noted to have impaired renal function. Her
urine sediment reveals red cell casts. The
results of immunological investigations are as
follows: serum IgG 6.5 g/L (normal range 6-13),
IgA 1.5 g/L (normal range 0.8-4.0), IgM 5.7 g/L
(normal range 0.4-2.0), serum electrophoresis
shows faint band in gamma region,
complement C3 1.02 g/L (normal range 0.751.65), complement C4 <0.02 g/L (normal range
0.20-0.65) and rheumatoid factor 894 IU/L
(normal range <40).
Which of the following investigations is likely
to be most important in making a definitive
diagnosis?
A Antineutrophil cytoplasmic antibodies
.

B Antinuclear antibodies
.
C Antiglomerular basement membrane
. antibodies
D Antimyeloperoxidase antibodies
.
E. Cryoglobulins
Correct Answer: E
Comment
The combination of a markedly low C4 (with
normal C3), elevated rheumatoid factor,
elevated serum IgM on a background of active
urinary sediment and a history of blood
transfusion is highly suggestive of hepatitis Cassociated cryoglobulinaemic vasculitis. Of the
investigations listed, cryoglobulins are the single
most important test in establishing a definitive
diagnosis in this patient.
43. A patient with psoriatic arthritis has active
joints and troublesome plaque psoriasis. Which
of the following will improve both the joint and
skin problems?
A Sulphasalazine
.
B Hydroxychloroqui
. ne
C Gold

.
D Methotrexate
.
E. Penicillamine.
Correct Answer: D
Comment
Hydroxychloroquine can exacerbate psoriasis.
Sulphasalazine tends to only improve joint symptoms
and not improve the psoriasis. Gold and
penicillamine are not commonly used to treat this
condition.
44. A 45-year-old woman developed Raynaud's
2 years ago. She now complains of
breathlessness and skin tightness affecting her
fingers. A high-resolution CT scan shows
evidence of pulmonary fibrosis.
Which of the following tests is most likely to be
positive?
A Anticentromere antibody
.
B Anti-double-stranded DNA
. antibody
C Anti-Ro antibody
.
D
. Anti-Scl-70 antibody

E. Anti-Jo-1 antibody
Correct Answer: D
Comment
This patient has clinical symptoms suggestive of
diffuse cutaneous systemic sclerosis. Pulmonary
fibrosis and anti-Scl-70 are more common in patients
with diffuse disease. Anticentromere antibody is
associated with limited cutaneous systemic sclerosis.
Anti-double-stranded DNA antibody is associated
with systemic lupus erythematosus. Anti-Ro antibody
is associated with lupus and primary Sjgren's
syndrome. Anti-Jo-1 is associated with polymyositis,
particularly in patients with inflammatory lung
disease.

45. A 58-year-old woman is referred with pain


and stiffness in her hands and knees. She has a
few patches of psoriasis on her arms. Her
hands are shown in the picture. What are the
two most likely diagnoses?
A Gout
.
B Nodal osteoarthritis
.
C Pseudogout
.
D Systemic sclerosis
.
E. Psoriatic arthritis
F. Systemic lupus
erythematosus
G Ankylosing spondylitis
.
H Rheumatoid arthritis
.
I. Reactive arthritis
J. SAPHO syndrome.
Correct Answer: BE
Comment
Swelling of the distal interphalangeal joints really
only occurs in nodal osteoarthritis and one of the
forms of psoriatic arthritis, which is usually easily
distinguished by nail involvement (not present in this

case).
Involvement of the base of the thumb is also
pathognomonic of nodal osteoarthritis, giving the
thumb base a characteristically square appearance.
46. A 35-year-old man is referred to you for
investigation of recurrent infection. He has had
frequent respiratory tract infections for the
past 5 years, requiring 4-5 courses of
antibiotics each winter. A month previously he
was admitted with pneumococcal pneumonia.
Two months prior to that he had sinus surgery.
However, this did not improve his recurrent
sinusitis. Which of the following is not in the
differential diagnosis?
A Antibody deficiency
.
B HIV infection
.
C Bronchiectasis secondary to recurrent
. infection
D Complement C6 deficiency
.
E. Smoking 5 cigarettes per day.
Correct Answer: D
Comment
Antibody deficiency is typically associated with
respiratory tract infections. Ask about diarrhoea and
bacterial skin infections which are also common. Take
a careful drug history and bear in mind the possibility

of lymphoproliferative disease. HIV infection,


although primarily associated with CD4 loss, also
results in antibody dysfunction, resulting in recurrent
respiratory tract infections in some patients. Ask
about features of cellular immune deficiency (oral
candida, herpes simplex and zoster, warts). Ask
about risk factors.
Recurrent bacterial chest infections, whatever their
cause, will eventually result in bronchiectasis,
hence the importance of early diagnosis and
treatment. Terminal complement deficiencies (C59) are extremely rare. Patients are well but have
increased susceptibility to neisserial infection.
Smoking causes ciliary paralysis. The resultant
mucociliary dysfunction is a common (and reversible)
cause of recurrent respiratory tract infection.
47. An infant presents with widespread
lymphadenopathy, hepatosplenomegaly and
fever one month after receiving a BCG vaccine.
A lymph node biopsy reveals acid-fast bacilli.
Which 2 of the following components of the
immune system is it most important to assess?
A T-cells
.
B B-cells
.
C C3
.

D Neutrophils
.
E. Platlets
F. IL-10
G IFN-gamma
.
H Mannan-binding. protein
I. C4
J. IL-2.
Correct Answer: AG
Comment
Disseminated BCG infection is uncommon and
suggests an underlying immune deficiency. The
immune response to mycobacterial infection is
characterised by granulomas composed of T-cells
and macrophages, whose development
critically involves interferon gamma, IL-12 and
TNF. Patients with inherited deficiencies of IL-12 and
IFN-gamma are predisposed to disseminated BCG
infection and tuberculosis is a recognised risk in
patients treated with anti-TNF therapy.
48. A 65-year-old man is admitted as an
emergency with a very hot, swollen left knee.
On examination he is unwell and pyrexial. He
has marked loss of range of movement,
secondary to pain. You have aspirated his knee

and have sent the purulent-looking fluid for


microscopy. Which of the following is true?
A He is likely to have acquired this infection
. after an arthroscopy or arthrocentesis.
B Seventy to eighty percent of cases will have
. an accompanying bacteraemia.
C Results of the culture should be awaited
. before commencing antibiotics.
D Antibiotics should cover beta-haemolytic
. streptococcus and staphylococcus
infections.
E. A plain x-ray of the knee will confirm the
diagnosis.
Correct Answer: D
Comment
This is likely to be septic arthritis. Obviously,
examination of the synovial fluid will help to exclude
differentials such as gout and pseudogout. The most
likely organisms are beta-haemolytic
streptococci (20%) and staphlococci (70%).
Antibiotics should be started empirically to cover
these if the clinical suspicion is high. Ideally these
should be intravenous for 2 weeks and then oral
for 4 weeks. Arthroscopy is a risk factor for septic
arthritis, but is rare. Fifty percent of cases will
have an associated bacteraemia. Early x-rays are
almost always normal.
49. A 28-year-old man is has recently been
discharged from hospital after treatment for

pneumococcal pneumonia. He has had


repeated courses of antibiotics for sinus, ear
and lower respiratory tract infections, and had
sinus surgery the previous year. He is a life
long non-smoker and is not on medication. His
blood count prior to discharge was normal.
In the absence of further clues in the history or
examination, which single blood test is the
most important?
A HIV antibody test
.
B Pneumococcal
. antibodies
C Immunoglobulin levels
.
D Liver function tests
.
E. IgG subclass levels.
Correct Answer: C
Comment
Hypogammaglobulinaemia is associated with
recurrent bacterial infections, most commonly of the
respiratory tract. Delay of several years prior to
diagnosis is usual, with associated morbidity. Patients
with low immunoglobulin levels and recurrent
infections should be treated with immunoglobulin
replacement. More minor antibody defects, such as
IgG subclass or specific antibody (to pneumococcus)

defects can often be treated with appropriate


vaccinations and/ or prophylactic antibiotics.
50. A 16-year-old boy is admitted to the A&E
department after collapsing following a wasp
sting. His blood pressure is recorded at 90/50
mmHg and examination reveals a widespread
urticarial rash. His blood pressure and rash
respond to 0.5mg i.m. adrenaline, 100mg
hydrocortisone i.v. and 10 mg
chlorpheniramine i.v. After being seen in clinic
two weeks later, which two of the following
tests would be most clinically useful?
A IgE
.
B Wasp venom radioallergensobent test
. (RAST)
C Serum tryptase
.
D Antinuclear antibodies (ANA)
.
E. Thyroid-stimulating hormone (TSH)
F. Wasp venom skin prick test
G Urinary catecholamines
.
H Liver function test (LFT)
.
I. Eosinophil count
J. Lymphocyte subsets

Correct Answer: BF
Comment
This patient has suffered an anaphylactic reaction
to bee venom. A rise in serum tryptase (if taken
within 6 hours) can help in identifying cases of
anaphylaxis where there is diagnostic uncertainty.
However, in this patient the classical features and
temporal relationship to a bee sting establish the
diagnosis clinically. He should be tested for venom
specific IgE by both skin prick testing and RAST; if
either are positive his anaphylactic risk for a further
wasp sting can be dramatically lowered by venom
immunotherapy
51. A 50-year-old woman presents to casualty
with a 2-year history of recurrent angioedema.
Her clinical history fails to reveal an underlying
trigger. Her serum complement profile is as
follows: serum C3 1.2 g/l (ref range 0.75-1.65),
serum C4 <0.02 g/l (ref range 0.2-0.6). The
most likely diagnosis is:
A food allergy
.
B drug allergy
.
C Idiopathic
. angioedema
D C1 inhibitor
. deficiency

E. venom allergy.
Correct Answer: D
Comment
Uncontrolled activation of the classical complement
pathway leading to a marked reduction in serum C4
levels is a hallmark of C1 inhibitor deficiency. In
the absence of a family history, it is possible that this
represents a new mutation. Alternatively, the
possibility of acquired C1 inhibitor deficiency due to
lymphoproliferative disease should be considered.
52. A 77-year-old man presents with persistent
head ache and progressive deafness. On
examination the patient has frontal bossing of
the forehead and conductive deafness, more
severe in the right ear. His serum alkaline
phosphatase is significantly raised at 870 u/L.
Which of the following statements is most
accurate about this disease?
A It usually affects a single bone.
.
B The skull is the most commonly affected
. bone.
C Bone pain is the most common presenting
. feature.
D Hearing loss is often due to involvement of
. the middle ear ossicles leading to conductive
deafness.

E. Bone pain is typically increased with rest


and on weight bearing.
Correct Answer: E
Comment
Paget's disease of bone is a focal disorder of bone
remodelling characterized by an increase in the
number and size of osteoclasts in affected skeletal
sites while the rest of the skeleton is spared. Pagets
disease most commonly involves the axial skeleton,
the pelvis being the most common, but it can
affect any area. In the majority of patients, the
disease affects at least two bones, but in one third of
patients only one bone is affected. In the skull, the
8th nerve can be compressed, resulting in hearing
loss. This is one of the more common complaints,
being present in 37% of respondents in a recent
survey of 2000 patients with Paget's disease . Other
causes of hearing loss include pagetic involvement of
the middle ear ossicles, which dampens the motion
of these ossicles.
Unlike osteoarthritis, pagetic bone pain usually
increases with rest, on weight bearing, when the
limbs are warmed, and at night. An estimated 70% of
patients who have Pagets disease have no
symptoms. The diagnosis is typically found
incidentally on radiographs and laboratory
investigations.
53. A young patient is attending the
rheumatology monitoring clinic. She develops

leucopaenia. Which of the following is unlikely


to cause this abnormality?
A Azathioprine
.
B Methotrexate
.
C Systemic lupus erythematosus
. (SLE)
D Cyclosporin A
.
E. Etanercept.
Correct Answer: D
Comment
Patients taking azathioprine, methotrexate and
etanercept require regular monitoring of the full
blood count to exclude marrow suppression.
Patients with SLE often have a low white cell count.
54. A 72-year-old man suffers a stroke causing
left sided weakness. He has very little
voluntary movement of the left arm and needs
assistance to transfer. Whilst on the
rehabilitation ward he complains of significant
pain around his left shoulder. Which of the
following statements is correct?
A Poor handling and positioning of the left arm
. by staff are unlikely to have contributed to
the pain
B Shoulder pain causes distress but does not

affect outcome

C A radiograph of the shoulder is not


. necessary
D Treatment with simple analgesics is a
. sensible initial approach
E. Intra-articular steroids are likely to
exacerbate the problem.
Correct Answer: D
Comment
Shoulder pain is a common complication of
stroke. The aetiology is thought to be multifactorial
and includes poor handling of the affected arm
causing trauma to the shoulder structures,
glenohumeral subluxation, abnormal tone (spasticity
and flaccidity) and pain due to the stroke itself
(central post-stroke pain). It is associated with
prolonged hospital stay and poor recovery of arm
function. A radiograph should be performed to
exclude fracture or dislocation. Treatment usually
starts with simple analgesics or non-steroidals.
Physiotherapists may treat subluxation with
strapping.If the pain persists intra-articular steroid
injections and TENS may help.

56. A 78-year-old woman found it much easier


to manage after the occupational therapist
provides several aids and appliances for use in
the kitchen. From their appearance, the
problem affecting her hands (see image) is:
A rheumatoid
. arthritis
B nodal
. osteoarthritis
C psoriatic
. arthropathy
D tophaceous gout
.
E. Charcot joints.
Correct Answer: C
Comment

This is the classic appearance of arthritis mutilans


in psoriatic arthropathy, with a deforming destructive
arthritis of the small joints. Osteolysis results in
shortening of some of the digits. The pattern is not
that of rheumatoid arthritis and the wrists appear to
be spared. There is too much destruction for
osteoarthritis. The appearance of gout may be
similar, though the tophi would be easier to see in
life (as opposed to a photo). Charcot joints
(neuropathic arthropathy) usually affect the foot and
ankle.
57. A 56-year-old man presents with recurrent
attacks of polyarticular gout despite treatment
with allopurinol 300 mg daily for the last year.
He is adamant that his compliance with his
drug treatment regimen is good.
Which of the following factors is most likely to
explain his poor response to allopurinol?
A
. Concurrent treatment with
colchicine
B Fast metabolism of allopurinol
.
C High alcohol intake
.
D Poor urate clearance via the
. kidney
E. High dietary purine intake

Correct Answer: C
Comment
Persistently high alcohol consumption is a
common cause of poor response to allopurinol,
although the underlying mechanism of this is unclear.
B, D and E are plausible answers, but are less
important in practice. Most adults will respond to
allopurinol 300 mg daily, although a small proportion
will require 600 or even 900 mg daily. The aim of
treatment should be to suppress the serum urate
level to the lower end of the normal range or just
below.
58. A 68-year-old woman complained of
progressive pain in her left hip after slipping
on ice and sustaining a sub-capital fracture of
the left femoral neck hip. Following recovery
from a hemiarthroplasty, bone mineral density
(BMD) measurement of the opposite femur
confirmed the diagnosis of osteoporosis with a
T score of -2.67. She did not wish to take
oestrogen replacement therapy and was given
alendronate 70 mg weekly. Which of the
following statements is true?
A Normally the peak bone mass in a woman is
. achieved just before the menopause.
B Plain radiographs are very sensitive in
. identifying minor reduction in bone mineral
density and hence a very useful screening
test.
C The T score compares a patients BMD with

the mean value for young, healthy adults of


the same sex.

D A high serum phosphate level may be


. suggestive of chronic alcoholism.
E. Bisphosphonates such as alendronate act by
stimulating bone formation.
Correct Answer: C
Comment
Bone mass increases during childhood and
adolescence and peaks between the ages of 20 and
30 years. Peak bone mass is influenced by age, sex,
genetic factors, hormonal status, exercise, and
calcium intake. As women in general have a lower
peak bone mass than men, it is understandable why
osteoporosis is predominantly seem in women.
Plain radiographs are not sensitive enough to
diagnose osteoporosis. Results of bone mineral
density (BMD) tests are typically reported as T
scores and Z scores.

The T score compares a patient's BMD with the


mean value for young, healthy adults of the
same sex.

The Z score compares a patient's BMD with the


mean value for persons of the same age and
sex.
Both scores are expressed in terms of standard
deviations from the mean. The World Health
Organization has defined osteoporosis as a BMD of at
least 2.5 standard deviations (SD) below the mean

value in young normal adults (i.e. T<=-2.5).


Chronic alcoholism is another important cause of low
serum phosphate level, in which case the liver
enzymes will be elevated. All bisphosphonates act
similarly on bone in binding permanently to
mineralized bone surfaces and inhibiting
osteoclastic activity. Thus, they inhibit bone
resorption and less bone is degraded during the
remodelling cycle. They do not stimulate bone
formation.
59. A 68-year-old man has lost weight. His
alkaline phosphatase is raised at 290 U/L
(normal range 35-120). Plain radiographs show
sclerotic lesions of bone.
What is the likely diagnosis?
A Stomach cancer
.
B Prostate cancer
.
C Multiple
. myeloma
D Lung cancer
.
E. Osteomalacia
Correct Answer: B
Comment

Cancer of the prostate is typically associated


with sclerotic bone lesions in contrast to the lytic
lesions seen in multiple myeloma. Osteomalacia is
associated with Looser's zones.
60. A 33-year-old woman presents with a 6month history of Raynaud's phenomenon
affecting her hands and feet. She is previously
well and takes no medication.
Which two of the following clinical features or
investigation results are the strongest
predictors that she will develop a connective
tissue disease in the future?
A Age >25 years
.
B Abnormal nail-fold capillary
. microscopy
C Elevated erythrocyte sedimentation
. rate
D History of recurrent miscarriage
.
E. Family history of Raynaud's
F. Anaemia
G Strongly positive antinuclear
. antibody
H History of chilblains
.
I. Raised platelet count

J. Dry eyes and dry mouth


Correct Answer: BG
Comment
These factors are strongly predictive of a future
connective tissue disease (CTD), particularly
abnormal nail-fold capillaries. The likelihood of
developing a CTD also increases with age of onset of
Raynaud's, with a particularly high risk in those aged
over 35 years. All the other features apart from a
family history (which suggests primary Raynaud's)
are associated with CTD, but have not been shown to
have the same predictive value as B and G.
61. A 22-year-old man has chronic diarrhoea
and has had several episodes of sinusitis and
pneumonia. A diagnosis of common variable
immunodeficiency (CVID) is made. Which two
of the following are recognized long-term
complications of this disorder?
A Type II diabetes
.
B Bronchiectasis
.
C Hyperparathyroidi
. sm
D Atheroma
.
E. Epilepsy
F. Enteropathy

G Cardiomyopathy
.
H Chronic renal
. failure
I. Optic atrophy
J. Sacroileitis.
Correct Answer: BF
Comment
CVID patients suffer from recurrent pulmonary
infections, and resultant repeated structural lung
damage can progress to bronchiectasis.
Gastrointestinal symptoms are a common problem in
CVID with a significant proportion being attributed to
a celiac like enteropathy.
62. A 40-year-old woman presents with a 6month history of a purpuric rash on her legs,
non-specific joint pains and vague ill health.
Initial investigations reveal a positive
rheumatoid factor. The GP makes a tentative
diagnosis of rheumatoid arthritis and refers
her to hospital. Results of further
investigations are as follows: Urinalysis: red
cell casts, protein+, ANA 1/80, anti-DNA
negative, anti-ENA negative, serum C3 1.02 g/l
(NR 0.75-1.65), C4 <0.02 g/l (0.20-0.65), CRP
<5mg/l (NR <5), creatinine 145 (NR 50-140).
Which is the most likely diagnosis?
A Systemic lupus erythematosus
. (SLE)

B Rheumatoid arthritis
.
C Mixed cryoglobulinaemia
.
D Primary complement deficiency
.
E. C1 inhibitor deficiency.
Correct Answer: C
Comment
This lady has a hypocomplementaemic
glomerulonephritis (note active urine sediment) and
a purpuric rash accompanied by a positive
rheumatoid factor and an isolated positive ANA.
The combination of renal and skin involvement in this
setting with a markedly low C4 suggests mixed
cryoglobulinaemia. Her serology does not support
SLE.
The key investigations which would help confirm the
diagnosis would be a warm clotted sample for
cryoglobulins and a renal biopsy.
63. A 69-year-old man with rheumatoid
arthritis presents with increased
breathlessness and nose bleeds. Which of the
following treatments for rheumatoid arthritis
are unlikely to cause bone marrow
suppression?
A Gold

.
B Azathioprine
.
C Non-steroidal anti-inflammatory drugs
. (NSAIDs)
D Methotrexate
.
E. Sulphasalazine
F. Infliximab
G Leflunomide
.
H Cyclophosphamide
.
I. Penicillamine
J. Hydroxychloroquine
Correct Answer: CF
Comment
Many of the treatments for rheumatoid arthritis
cause bone marrow suppression. NSAIDs may be
used early in the disease and cause anaemia via
gastrointestinal ulceration. Infliximab has been
associated with multiple sclerosis-like demyelination,
the development of tuberculosis and worsening heart
failure.
64. A 70-year-old female has a 17-year history
of rheumatoid arthritis. She presents with

recurrent attacks of red congested eyes with a


sensation of grittiness. The most likely cause
of her red eyes is likely to be:
A scleritis
.
B episcleritis
.
C keratitis
.
D keratoconjunctivitis
. sicca
E. choroiditis.
Correct Answer: D
Comment
Approximately 25 % of patients with rheumatoid
arthritis (RA) will have ocular manifestations.
keratoconjunctivitis sicca, scleritis, episcleritis,
keratitis, peripheral corneal ulceration, and other less
common entities such as choroiditis, retinal
vasculitis, episcleral nodules, retinal detachments,
and macular edema.
Keratoconjunctivitis sicca, or dry eye syndrome,
is the most common ocular manifestation of RA and
has a reported prevalence of 15 to 25 %. The
patient reports a gritty sensation in the eyes .
Scleritis and episcleritis as causes of recurrent red
eyes are distinguished on the basis of anatomy and
appearance. Symptoms may be similar, but the pain

in scleritis is more evident and severe. Tenderness to


palpation of the globe can help differentiate the two.
After asking the patient to look down with eyelids
closed, the physician gently presses the globe.
Patients with scleritis have tenderness on palpation,
while those with episcleritis do not.. Unlike scleritis,
patients with episcleritis do not complain of blurred
vision or photophobia. The importance of correctly
diagnosing and distinguishing between scleritis and
episcleritis is based on the potential ocular and
systemic complications associated with scleritis.
Studies have shown that patients with RA-associated
scleritis have more widespread systemic disease and
a higher mortality rate than those episcleritis.
65. A 20-year-old man with common variable
antibody deficiency presents to the Emergency
Department with a 3-day history of cough
productive of green sputum. His temperature
is 37.5C, p 84/min, respiratory rate 12/min,
and breath sounds are vesicular. Chest
radiograph is unremarkable. Which two of the
following actions do you recommend
immediately?
A Prescribe a 14 day course of antibiotics
.
B Prescribe nebulised salbutamol
.
C Give an infusion of intravenous
. immunoglobulin

D Take a sputum sample to look for acid fast


. bacilli
E. Prescribe a 5-day course of antibiotics
F. Check his serum immunoglobulin levels
G Choose an antibiotic regimen suitable for
. possible pseudomonas infection
H Order a high resolution CT scan
.
I. Take a sputum sample for culture
J. Prescribe a 28 day course of antibiotics.
Correct Answer: AI
Comment
Antibody deficient patients need prompt treatment of
presumed bacterial infection. Treatment should be
continued for slightly longer than normal; 14 days for
an uncomplicated chest infection would be
appropriate. Most infections are caused by common
organisms such as haemophilus or
pneumococcus. Psuedomonas is unusual and
mycobacterial disease rare in common variable
immunodeficiency. Cultures are invaluable if there is
a poor response to treatment and for guiding future
antibiotic choices. For infections causing fever,
routine antibody replacement should be deferred for
24-48 hours until there is a clear response to
treatment, as adverse reactions are much more
common in the presence of fever.

67. A 65-year-old woman with a mitral valve


replacement presents to the Emergency
Department with pyrexia and fainting. She is
unwell, hypotensive, anaemic and pyrexial. She
has a vague history of suffering from a
reaction to penicillin in her childhood. After
taking blood cultures she is started on broadspectrum antibiotics. Cardiac valvular
vegetations are seen on echocardiography and
her blood grows methicillin-sensitive
Staphylococci the microbiologist suggests
naficillin as the most appropriate antibiotic,
but is concerned that she may have allergy to
beta lactam-based antibiotics.
Which of the following is most appropriate to
investigate her history of possible penicillin
allergy?
A serum tryptase
.
B skin prick test to
. penicillin
C serum penicillin specific
. IgE
D patch test to penicillin
.
E. serum IgE
Correct Answer: B
Comment

A history of penicillin allergy is relatively common in


hospital patients, and is in most not due to type I
hypersensitivity reactions. A diagnosis of
penicillin allergy crucially requires a detailed history
of the drug reaction, and can be confirmed by a
positive skin prick test to the major and minor
determinants of penicillin. Skin prick testing is carried
out if there is a clinical need for penicillin treatment
e.g. treatment of infective endocarditis. A patient is
unlikely to develop anaphylaxis with a negative
penicillin skin prick test. The detection of penicillin
specific IgE in the serum is unreliable.
68. A 36-year-old man presents with a 2-day
history of severe pain in the left knee. He has
recently returned from a holiday in Spain.
Examination shows low grade fever (37.5C)
and marked synovitis of the left knee with a
tense effusion. Which are the two most likely
diagnoses?
A Lyme disease
.
B Rheumatoid arthritis
.
C Pseudogout
.
D Gout
.
E. Septic arthritis
F. Osteosarcoma
G Osteoarthritis

.
H Reactive arthritis
.
I. Torn medial meniscus
J. Spontaneous
haemarthrosis.
Correct Answer: DH
Comment
Reactive arthritis, consequent upon either genital
chlamydia trachomatis or enteric infection, is the
most likely diagnosis. He is a little younger than
many patients presenting with gout, but this is not a
rare diagnosis in this age group, particularly if
alcohol intake is high. The important diagnosis to
exclude is septic arthritis, but this is a rare cause of
monoarthritis in this age group. Aspiration of synovial
fluid is the key diagnostic tool to differentiate
between these diagnoses.
69. A 65-year-old woman with primary
Sjgren's syndrome, which was diagnosed 1
year previously, is noted to have a persistently
elevated erythrocyte sedimentation rate (ESR)
of 90-100 mm/hour and a normal C-reactive
protein (CRP). At a routine outpatient visit it is
noted that her sicca symptoms are still
troublesome, but there are no other
abnormalities on clinical examination. The
following investigations are performed:
haemoglobin 12.5 g/dL (normal range 12-16),

white cell count 6.4 x 109/L (normal range 411), platelet count 320 x 109/L (normal range
150-400), ESR 98 mm/hour, CRP <6 mg/dL
(normal <6), serum IgG 42 g/L (normal range 613), IgA 8.4 g/L (normal range 0.8-4.0) and IgM
3.6 g/L (normal range 0.4-2.0), and serum
electrophoresis shows polyclonal
hypergammaglobulinaemia.
Her persistently elevated ESR is best explained
by which of the following?
A Development of lymphoma
.
B Poorly controlled sicca symptoms
.
C Positive antinuclear antibody
.
D Positive antibodies to Ro and La
. antigens
E. Polyclonal
hypergammaglobulinaemia
Correct Answer: E
Comment
Together with fibrinogen, serum
immunoglobulins are the major driving force
causing an elevation in erythrocyte
sedimentation rate (ESR). Any disorder associated
with a persistently elevated polyclonal
hypergammaglobulinaemia, such as primary
Sjgren's syndrome, is likely to be linked with a

persistently elevated ESR. Note that her C-reactive


protein (CRP) is normal, which reflects the fact that
CRP production is not influenced by changes in
serum immunoglobulin levels.
70. A 28-year-old woman presented with
fatigue and extreme tiredness. Physical
examination revealed facial skin rash and
tenderness across the small joints of the
hands. She was concerned that she might have
systemic lupus erythematosus (SLE). Which of
the following tests when NEGATIVE will
virtually exclude the diagnosis of SLE?
A Antinuclear antibody (ANA)
.
B Anti-double stranded DNA (anti ds. DNA)
C Anti-Sm antibodies
.
D Anti-histone antibodies
.
E. Anti-Ro/SSA antibodies.
Correct Answer: A
Comment
Almost all patients with SLE have a positive
ANA test result. The ANA test is sensitive but not
specific for SLE. A negative result argues strongly
against a diagnosis of active SLE, but does not
exclude the possibility of other autoimmune

diseases.
Antibodies to Sm antigen are highly specific for
a diagnosis of SLE (>99%). However, only about
25% of patients with SLE have anti-Sm antibodies.
Anti-DNA antibodies are diagnostic of SLE
(specificity >99%). However, only 60% of patients
with SLE will have these antibodies. Therefore,
absence of anti-DNA or anti-Sm antibodies should not
exclude SLE as a diagnosis. Anti-Ro/SS-A antibodies
are found in 30% of patients with SLE. Anti-histone
antibodies are identified in small proportion of SLE
patients. They are more often seen with druginduced lupus.

31. A 30-year-old man has a swollen wrist, low


back pain with early morning stiffness lasting 2
hours and mouth ulcers. He does not have a
rash. What is the most likely diagnosis?
A Rheumatoid arthritis (RA)
.
B Psoriatic arthritis
.
C Ankylosing spondylitis
.

D Gout
.
E. systemic lupus erythematosus
(SLE)
Correct Answer: C
Comment
The history is suggestive of a seronegative
spondyloarthropathy.
RA is classically a peripheral, symmetrical
polyarthritis.
Gout can affect the wrist but is unusual in a
young person.
Patients with SLE can experience mouth ulcers but
SLE is uncommon in young males.

32. A 52-year-old accountant, with a 6-year


history of Raynauds phenomenon presents to
her G.P. with a 4-month history of worsening
dysphagia. Incubation of her serum on Hep 2
cells reveals the following immunofluorescent
pattern (see image).
A Primary Raynauds disease
.
B systemic lupus erythematosus
. (SLE)
C Limited systemic sclerosis
.
D Rheumatoid arthritis
.
E. Sjogren's disease.
Correct Answer: C
Comment
Raynauds phenomenon is common, occurring in
4-15% of the general population and in the majority
of cases is not associated with connective tissue
disease. However, the detection of an anticentromere nuclear staining pattern has a high
specificity for limited systemic sclerosis,
especially in the context of dysphagia which is a
feature of the disease.
33. An 18-year-old student with known asthma
and peanut allergy collapses following a meal
in the hospital canteen. Strenuous attempts at
resuscitation are tragically unsuccessful. It is

thought that the most likely cause of his


collapse was an anaphylactic reaction to
hidden' nuts contained in the meal.
Which of the following mast cell-derived
mediators is it most useful to measure to
confirm the clinical suspicion of anaphylaxis?
A Mast cell - derived growth
. factor
B Chymase
.
C Interleukin-6
.
D Tryptase
.
E. Transforming growth factor
Correct Answer: D
Comment
Amongst the array of mediators released by mast
cells during anaphylaxis, tryptase has proven to be
a reliable marker of mast cell degranulation on
account of its stability, relatively long half-life and
ease of measurement. Consequently, an elevated
tryptase in the correct clinical context is a useful
surrogate marker of an anaphylactic/anaphylactoid
reaction.
34. A 60-year-old woman develops acute,
severe low thoracic back pain. A radiograph

shows a vertebral crush fracture. Which two of


the following reduce an individual's risk for
developing osteoporosis?
A Early menarche
.
B Early menopause
.
C Smoking
.
D High alcohol intake
.
E. Prolonged treatment with
corticosteroids
F. Rheumatoid arthritis
G Crohn's disease
.
H Asthma
.
I. Frequent walks
J. Obesity.
Correct Answer: AI
Comment
The most likely cause of a vertebral crush factor in a
woman of this age is obviously osteoporosis, but
local vertebral pathology - in particular secondary
tumours or myeloma - would need to be considered.

It is most likely that the cause of osteoporosis would


be postmenopausal in this woman, but many medical
factors influence the likelihood of this condition.
The following tests should be considered in a
patient presenting with an osteoporotic
fracture (although not all need to be performed in
all cases): full blood count, renal / liver / bone
chemistry, immunoglobulins and serum / urine
electrophoresis (to exclude myeloma), thyroid
function, testosterone (in men), investigations for
Cushing's syndrome.

35. This patient was trying to stand up. On


examination he has periorbital oedema with a

faint purple hue. The serum creatinine kinase


(CK) levels were 10-times the upper limit of
normal. Which of the following manifestations
is least likely to be associated with this
disorder?
A blue and white color change in the fingers
. on exposure to cold
B difficulty in swallowing liquids
.
C double vision on looking to the sides
.
D radial pulse of 40 beats per minute
.
E. persistent cough, haemoptysis and weight
loss of 12 kg in the last three months
Correct Answer: C
Comment
This patient is not able to stand up from a chair
unaided. This indicates proximal muscle
weakness. The skin rash and the raised levels of
muscle enzyme point towards an inflammatory
myopathy, namely dermatomyosytis (DM).
DM is easily recognized and diagnosed by a
characteristic rash:

a heliotrope rash (blue-purple discoloration) on


the upper eyelids, with oedema

a flat red rash on the face and upper trunk.

Erythema of the knuckles accompanied by a


raised, violaceous scaly eruption (Gottron's
sign) is also characteristic, and may precede or
follow muscle weakness. Raynaud's phenomenon
with characteristic color change (pallor, cyanosis and
rubor) on exposure to cold is encountered in patient
with DM, more often when there is overlap with other
connective tissue disease such as scleroderma or
mixed connective tissue disease.
Inflammation of the skeletal muscles of the
oropharynx and upper oesophagus lead to
dysphagia especially for liquids in the initial stages
of the disease which progresses to difficulty in
swallowing solids as the disease progresses.
Electrocardiographic evidence of conduction
defects and arrhythmias occur frequently in
childhood and adult DM, although overt clinical
symptoms are uncommon.
There is an increased incidence of underlying
malignancy in adult DM, ranging from 5% to 15%.
Carcinoma of the bronchus, ovaries and breast
are the most common.
Ocular muscles remain normal, even in
advanced, untreated cases, and if these muscles are
affected, the diagnosis of inflammatory myopathy
should be in doubt.
36. A 34-year-old woman presents to your
clinic complaining of cold hands, particularly in
the winter months. On examination, she has
cold dusky hands and a petechial rash.
Investigations are as follows: Hb 10.9 g/dL;
Wbc 4.2 109/L; Platelets 407 x 109/L; urea and
electrolytes - normal; liver function tests

- normal; albumin 36 g/L; globulin 90 g/L;


protein electrophoresis - polyclonal increase in
gammaglobulins; antinuclear antibodies
present (1/160, speckled pattern); complement
- C3 0.79 (NR 0.75-1.25), C4 <0.04 (NR 0.140.6). Which of the following statements is true?
A Active systemic lupus erythematous (SLE) is
. unlikely if DNA antibodies are present.
B A blood sample sent to the lab on ice may
. show cryoglobulins.
C Sjorgrens syndrome is unlikely if
. rheumatoid factor is present.
D Sjorgrens syndrome is likely if Ro and La
. extractable nuclear antigens are present.
E. Hepatitis C is unlikely in this case.
Correct Answer: D
Comment
The symptoms and signs and low C4 are
suggestive of cryoglobulinaemia.
Sludging of proteins at reduced temperatures (for
example hands on a cold day) can cause ischaemia
and sometimes vasculitis, particularly of skin or
kidneys.
Cryoglobulinaemia is commonly associated with
hepatitis C or connective tissue disease, such as
Sjorgren's syndrome. The positive ANA and high
globulins suggest Sjorgren's but could also be
associated with chronic infection, such as hepatitis C.

Since they precipitate at low temperatures,


cryoglobulins should always be transported to the lab
at 37C. Failure to do this will result in a false
negative result as the cryos will precipitate and be
removed with the clot.
37. A 69-year-old man with a 10-year history of
rheumatoid arthritis, controlled with diclofenac
and sulphasalazine has reported recurrent
heart burn and dyspepsia. His doctor asked
him to stop the dicofenac sodium and replaces
it with celecoxib (one of the COX 2 inhibitors).
Which of the following is true about COX 2
selective agents?
A They eliminate the chance of non-steroidal
. anti-inflammatory drugs (NSAID)-associated
dyspepsia.
B They are now regarded as the primary
. treatment for rheumatoid arthritis.
C They are more effective than ordinary
. NSAIDs.
D Because of their potent antiplatelet effect
. the addition of low dose aspirin for
ischaemic heart disease prophylaxis is not
necessary.
E. They can cause acute renal failure when
used in patients with impaired renal
function.
Correct Answer: E
Comment

NSAIDs inhibit prostaglandin synthesis by inhibiting


the enzyme cyclooxygenase (COX). The existence of
two different forms of COX isoenzymes (COX-1 and
COX-2) has now been recognised. COX-1 has a
constitutive 'housekeeping' role and COX-2
upregulation is associated with inflammation. The
COX-2-selective NSAIDs are not more effective than
ordinary NSAIDs, they are not the primary treatment
for rheumatoid arthritis, and they do not eliminate
the chance of NSAID-associated dyspepsia. Unlike
aspirin and most traditional NSAIDs, they have no
antiplatelet effect, so they are not cardioprotective.
In clinical trials COX-2 selective NSAIDS reduced the
risk of developing significant upper gastrointestinal
complications in the first six months. However, the
risk of GI bleeding from nonselective NSAIDs is low an annual incidence of 1% to 3% - and the COX-2selective NSAIDs are costly. Therefore, it may be best
to reserve these drugs for patients at high-risk
patients to include those over 65 years, those
already using medications known to increase the
likelihood of upper gastrointestinal adverse events,
those with serious co-morbidity, and those requiring
prolonged use of maximum recommended doses of
standard NSAIDs.
All NSAIDs, including the COX-2 selective agents, can
cause renal toxicity, such as acute renal failure,
nephrotic syndrome, and acute interstitial nephritis.
Acute renal failure is a concern particularly in
patients with pre-existing renal dysfunction. NSAIDs
should be avoided also in patients with cirrhosis and
ascites or congestive heart failure. Periodic

monitoring of renal function is recommended,


particularly in elderly patients.
38. A 36-year old woman is referred with a 1year history of muscle pain, tiredness and
sleep disturbance. She denies fever, weight
loss and arthralgia. Examination reveals
tenderness over her occiput, trapezius and
lumbar area. Her blood results show a normal
ESR, CRP, FBC, a weakly positive ANA 1:80 and
normal complement. Which is the most likely
diagnosis?
A Polymyositis
.
B System lupus erythematous
. (SLE)
C Sjogrens syndrome
.
D Polymyalgia rheumatica
.
E. Fibromyalgia
Correct Answer: E
Comment
The lack of constitutional symptoms, normal
inflammatory markers and normal examination, apart
from evidence of tender points, make an
inflammatory rheumatological disease unlikely. The
presence of tender points, history of muscle pain and
sleep disturbance are suggestive of fibromyalgia
a non-inflammatory pain disorder.

39. A 25-year-old woman with a history of 3second trimester fetal losses is planning a
fourth pregnancy. She has evidence of the
primary anti-phospholipid syndrome (strongly
positive cardiolipin antibody, positive lupus
anticoagulant but no evidence of lupus). Which
of the following treatment regimens offer her
the best chance of having a successful
pregnancy?
A Steroids alone
.
B Steroids combined with low-dose aspirin
.
C Low-dose aspirin alone
.
D Low-dose aspirin combined with low
. molecular weight heparin
E. Intravenous immunoglobulin.
Correct Answer: D
Comment
Of the above treatment options, low-dose aspirin
combined with low molecular weight heparin offers
her the best chance of having a successful
pregnancy.
40. You are called to A&E to review a 28-yearold woman with pleuritic chest pain. Which of

the following statements concerning this


patient are true?
A Low white count is consistent with systemic
. lupus erythematosus (SLE)
B Low white count is not consistent with
. tuberculosis.
C The presence of anticardiolipin antibodies
. IgG 17 GPLU/ml (NR <14), IgM 21 MPLU/ml
(NR<10) suggests that she may have a
pulmonary embolus.
D The presence of antinuclear antibodies at a
. titre of 1/80 suggests that she may have a
connective tissue disease.
E. The presence of antinuclear cytoplasmic
antibodies (cANCA) at a titre of 1 in 20
suggests that Wegener's granulomatosis is
likely.
Correct Answer: A
Comment
Active SLE and tuberculosis are both
associated with leukopenia. Low titre antinuclear
antibodies (in the absence of DNA or ENA),
antineutrophil cytoplasmic antibodies (ANCA) and
anticardiolipin antibodies are non-specific and are
commonly found in the presence of infection.
41. A 66-year-old woman has a 15-year history
of deforming rheumatoid arthritis (RA). She is
maintained on D-Penicillamine. Two weeks ago
she noticed increased difficulty in climbing

stairs and three days before admission she was


unable to comb her hair or feed herself.
Neurological assessment is difficult
but reveals generalised grade 3/5 weakness,
and the Babiniski sign is positive bilaterally.
Which one of the following tests is most likely
to reveal the diagnosis?
A Plain radiograph of the cervical spine
.
B Electromyography (EMG)
.
C Nerve conduction study (NCS)
.
D Isotope bone scan
.
E. Magnetic resonance imaging (MRI) of the
cervical spine
Correct Answer: E
Comment
The recent limb weakness and the presence of
pyramidal signs in the legs in patients with RA are
highly suggestive of spinal cord lesion at the
cervical or thoracic region.
RA primarily affects the cervical spine; affliction of
the thoracic or lumbar spine is rare. The anatomical
abnormalities occur as a consequence of the
destruction of synovial joints, ligaments, and bone.
Atlantoaxial subluxation (AAS) is the most
common. Patients may experience weakness,

decreased endurance, gait difficulty, paresthesias of


the hands, and loss of fine dexterity. Multiple
neurological signs may be elicited on physical
examination, including diffuse hyperreflexia, lower
extremity spasticity, a spastic gait, and Babinski's
sign.
Although plain radiography of the cervical spine is
regarded as the initial imaging assessment tool for
neck pain in patient with RA, those with symptoms or
signs of cord compression should undergo immediate
MRI and be sent for surgical consultation. MRI is
considered the most sensitive imaging modality for
diagnosing spinal cord compression, evaluating its
extent, and assessing soft tissues (panus) as well as
bone destruction.
D-Penicillamine causes various neuromuscular
disorders including myathenia gravis.
There is no clinical evidence for primary muscle
disease or peripheral neuropathy in this patien,
hence EMG or NCS are not necessary.
Isotope bone scan may identify increased activity in
the cervical region, but this is not specific and the
investigation has little role in investigation of
suspected spinal cord compression.
42. A 70-year-old woman with a history of
blood transfusion in the early 1980s presents
with a 10-month history of malaise and is
noted to have impaired renal function. Her
urine sediment reveals red cell casts. The
results of immunological investigations are as
follows: serum IgG 6.5 g/L (normal range 6-13),

IgA 1.5 g/L (normal range 0.8-4.0), IgM 5.7 g/L


(normal range 0.4-2.0), serum electrophoresis
shows faint band in gamma region,
complement C3 1.02 g/L (normal range 0.751.65), complement C4 <0.02 g/L (normal range
0.20-0.65) and rheumatoid factor 894 IU/L
(normal range <40).
Which of the following investigations is likely
to be most important in making a definitive
diagnosis?
A Antineutrophil cytoplasmic antibodies
.
B Antinuclear antibodies
.
C Antiglomerular basement membrane
. antibodies
D Antimyeloperoxidase antibodies
.
E. Cryoglobulins
Correct Answer: E
Comment
The combination of a markedly low C4 (with
normal C3), elevated rheumatoid factor,
elevated serum IgM on a background of active
urinary sediment and a history of blood
transfusion is highly suggestive of hepatitis Cassociated cryoglobulinaemic vasculitis. Of the
investigations listed, cryoglobulins are the single

most important test in establishing a definitive


diagnosis in this patient.
43. A patient with psoriatic arthritis has active
joints and troublesome plaque psoriasis. Which
of the following will improve both the joint and
skin problems?
A Sulphasalazine
.
B Hydroxychloroqui
. ne
C Gold
.
D Methotrexate
.
E. Penicillamine.
Correct Answer: D
Comment
Hydroxychloroquine can exacerbate psoriasis.
Sulphasalazine tends to only improve joint symptoms
and not improve the psoriasis. Gold and
penicillamine are not commonly used to treat this
condition.
44. A 45-year-old woman developed Raynaud's
2 years ago. She now complains of
breathlessness and skin tightness affecting her
fingers. A high-resolution CT scan shows
evidence of pulmonary fibrosis.

Which of the following tests is most likely to be


positive?
A Anticentromere antibody
.
B Anti-double-stranded DNA
. antibody
C Anti-Ro antibody
.
D
. Anti-Scl-70 antibody
E. Anti-Jo-1 antibody
Correct Answer: D
Comment
This patient has clinical symptoms suggestive of
diffuse cutaneous systemic sclerosis. Pulmonary
fibrosis and anti-Scl-70 are more common in patients
with diffuse disease. Anticentromere antibody is
associated with limited cutaneous systemic sclerosis.
Anti-double-stranded DNA antibody is associated
with systemic lupus erythematosus. Anti-Ro antibody
is associated with lupus and primary Sjgren's
syndrome. Anti-Jo-1 is associated with polymyositis,
particularly in patients with inflammatory lung
disease.

45. A 58-year-old woman is referred with pain


and stiffness in her hands and knees. She has a
few patches of psoriasis on her arms. Her
hands are shown in the picture. What are the
two most likely diagnoses?
A Gout
.
B Nodal osteoarthritis
.
C Pseudogout
.
D Systemic sclerosis
.
E. Psoriatic arthritis
F. Systemic lupus
erythematosus

G Ankylosing spondylitis
.
H Rheumatoid arthritis
.
I. Reactive arthritis
J. SAPHO syndrome.
Correct Answer: BE
Comment
Swelling of the distal interphalangeal joints really
only occurs in nodal osteoarthritis and one of the
forms of psoriatic arthritis, which is usually easily
distinguished by nail involvement (not present in this
case).
Involvement of the base of the thumb is also
pathognomonic of nodal osteoarthritis, giving the
thumb base a characteristically square appearance.
46. A 35-year-old man is referred to you for
investigation of recurrent infection. He has had
frequent respiratory tract infections for the
past 5 years, requiring 4-5 courses of
antibiotics each winter. A month previously he
was admitted with pneumococcal pneumonia.
Two months prior to that he had sinus surgery.
However, this did not improve his recurrent
sinusitis. Which of the following is not in the
differential diagnosis?
A Antibody deficiency
.
B HIV infection

.
C Bronchiectasis secondary to recurrent
. infection
D Complement C6 deficiency
.
E. Smoking 5 cigarettes per day.
Correct Answer: D
Comment
Antibody deficiency is typically associated with
respiratory tract infections. Ask about diarrhoea and
bacterial skin infections which are also common. Take
a careful drug history and bear in mind the possibility
of lymphoproliferative disease. HIV infection,
although primarily associated with CD4 loss, also
results in antibody dysfunction, resulting in recurrent
respiratory tract infections in some patients. Ask
about features of cellular immune deficiency (oral
candida, herpes simplex and zoster, warts). Ask
about risk factors.
Recurrent bacterial chest infections, whatever their
cause, will eventually result in bronchiectasis,
hence the importance of early diagnosis and
treatment. Terminal complement deficiencies (C59) are extremely rare. Patients are well but have
increased susceptibility to neisserial infection.
Smoking causes ciliary paralysis. The resultant
mucociliary dysfunction is a common (and reversible)
cause of recurrent respiratory tract infection.

47. An infant presents with widespread


lymphadenopathy, hepatosplenomegaly and
fever one month after receiving a BCG vaccine.
A lymph node biopsy reveals acid-fast bacilli.
Which 2 of the following components of the
immune system is it most important to assess?
A T-cells
.
B B-cells
.
C C3
.
D Neutrophils
.
E. Platlets
F. IL-10
G IFN-gamma
.
H Mannan-binding. protein
I. C4
J. IL-2.
Correct Answer: AG
Comment
Disseminated BCG infection is uncommon and
suggests an underlying immune deficiency. The
immune response to mycobacterial infection is

characterised by granulomas composed of T-cells


and macrophages, whose development
critically involves interferon gamma, IL-12 and
TNF. Patients with inherited deficiencies of IL-12 and
IFN-gamma are predisposed to disseminated BCG
infection and tuberculosis is a recognised risk in
patients treated with anti-TNF therapy.
48. A 65-year-old man is admitted as an
emergency with a very hot, swollen left knee.
On examination he is unwell and pyrexial. He
has marked loss of range of movement,
secondary to pain. You have aspirated his knee
and have sent the purulent-looking fluid for
microscopy. Which of the following is true?
A He is likely to have acquired this infection
. after an arthroscopy or arthrocentesis.
B Seventy to eighty percent of cases will have
. an accompanying bacteraemia.
C Results of the culture should be awaited
. before commencing antibiotics.
D Antibiotics should cover beta-haemolytic
. streptococcus and staphylococcus
infections.
E. A plain x-ray of the knee will confirm the
diagnosis.
Correct Answer: D
Comment
This is likely to be septic arthritis. Obviously,
examination of the synovial fluid will help to exclude

differentials such as gout and pseudogout. The most


likely organisms are beta-haemolytic
streptococci (20%) and staphlococci (70%).
Antibiotics should be started empirically to cover
these if the clinical suspicion is high. Ideally these
should be intravenous for 2 weeks and then oral
for 4 weeks. Arthroscopy is a risk factor for septic
arthritis, but is rare. Fifty percent of cases will
have an associated bacteraemia. Early x-rays are
almost always normal.
49. A 28-year-old man is has recently been
discharged from hospital after treatment for
pneumococcal pneumonia. He has had
repeated courses of antibiotics for sinus, ear
and lower respiratory tract infections, and had
sinus surgery the previous year. He is a life
long non-smoker and is not on medication. His
blood count prior to discharge was normal.
In the absence of further clues in the history or
examination, which single blood test is the
most important?
A HIV antibody test
.
B Pneumococcal
. antibodies
C Immunoglobulin levels
.
D Liver function tests
.

E. IgG subclass levels.


Correct Answer: C
Comment
Hypogammaglobulinaemia is associated with
recurrent bacterial infections, most commonly of the
respiratory tract. Delay of several years prior to
diagnosis is usual, with associated morbidity. Patients
with low immunoglobulin levels and recurrent
infections should be treated with immunoglobulin
replacement. More minor antibody defects, such as
IgG subclass or specific antibody (to pneumococcus)
defects can often be treated with appropriate
vaccinations and/ or prophylactic antibiotics.
50. A 16-year-old boy is admitted to the A&E
department after collapsing following a wasp
sting. His blood pressure is recorded at 90/50
mmHg and examination reveals a widespread
urticarial rash. His blood pressure and rash
respond to 0.5mg i.m. adrenaline, 100mg
hydrocortisone i.v. and 10 mg
chlorpheniramine i.v. After being seen in clinic
two weeks later, which two of the following
tests would be most clinically useful?
A IgE
.
B Wasp venom radioallergensobent test
. (RAST)
C Serum tryptase
.

D Antinuclear antibodies (ANA)


.
E. Thyroid-stimulating hormone (TSH)
F. Wasp venom skin prick test
G Urinary catecholamines
.
H Liver function test (LFT)
.
I. Eosinophil count
J. Lymphocyte subsets
Correct Answer: BF
Comment
This patient has suffered an anaphylactic reaction
to bee venom. A rise in serum tryptase (if taken
within 6 hours) can help in identifying cases of
anaphylaxis where there is diagnostic uncertainty.
However, in this patient the classical features and
temporal relationship to a bee sting establish the
diagnosis clinically. He should be tested for venom
specific IgE by both skin prick testing and RAST; if
either are positive his anaphylactic risk for a further
wasp sting can be dramatically lowered by venom
immunotherapy
51. A 50-year-old woman presents to casualty
with a 2-year history of recurrent angioedema.
Her clinical history fails to reveal an underlying
trigger. Her serum complement profile is as

follows: serum C3 1.2 g/l (ref range 0.75-1.65),


serum C4 <0.02 g/l (ref range 0.2-0.6). The
most likely diagnosis is:
A food allergy
.
B drug allergy
.
C Idiopathic
. angioedema
D C1 inhibitor
. deficiency
E. venom allergy.
Correct Answer: D
Comment
Uncontrolled activation of the classical complement
pathway leading to a marked reduction in serum C4
levels is a hallmark of C1 inhibitor deficiency. In
the absence of a family history, it is possible that this
represents a new mutation. Alternatively, the
possibility of acquired C1 inhibitor deficiency due to
lymphoproliferative disease should be considered.
52. A 77-year-old man presents with persistent
head ache and progressive deafness. On
examination the patient has frontal bossing of
the forehead and conductive deafness, more
severe in the right ear. His serum alkaline
phosphatase is significantly raised at 870 u/L.

Which of the following statements is most


accurate about this disease?
A It usually affects a single bone.
.
B The skull is the most commonly affected
. bone.
C Bone pain is the most common presenting
. feature.
D Hearing loss is often due to involvement of
. the middle ear ossicles leading to conductive
deafness.
E. Bone pain is typically increased with rest
and on weight bearing.
Correct Answer: E
Comment
Paget's disease of bone is a focal disorder of bone
remodelling characterized by an increase in the
number and size of osteoclasts in affected skeletal
sites while the rest of the skeleton is spared. Pagets
disease most commonly involves the axial skeleton,
the pelvis being the most common, but it can
affect any area. In the majority of patients, the
disease affects at least two bones, but in one third of
patients only one bone is affected. In the skull, the
8th nerve can be compressed, resulting in hearing
loss. This is one of the more common complaints,
being present in 37% of respondents in a recent
survey of 2000 patients with Paget's disease . Other
causes of hearing loss include pagetic involvement of

the middle ear ossicles, which dampens the motion


of these ossicles.
Unlike osteoarthritis, pagetic bone pain usually
increases with rest, on weight bearing, when the
limbs are warmed, and at night. An estimated 70% of
patients who have Pagets disease have no
symptoms. The diagnosis is typically found
incidentally on radiographs and laboratory
investigations.
53. A young patient is attending the
rheumatology monitoring clinic. She develops
leucopaenia. Which of the following is unlikely
to cause this abnormality?
A Azathioprine
.
B Methotrexate
.
C Systemic lupus erythematosus
. (SLE)
D Cyclosporin A
.
E. Etanercept.
Correct Answer: D
Comment
Patients taking azathioprine, methotrexate and
etanercept require regular monitoring of the full
blood count to exclude marrow suppression.
Patients with SLE often have a low white cell count.

54. A 72-year-old man suffers a stroke causing


left sided weakness. He has very little
voluntary movement of the left arm and needs
assistance to transfer. Whilst on the
rehabilitation ward he complains of significant
pain around his left shoulder. Which of the
following statements is correct?
A Poor handling and positioning of the left arm
. by staff are unlikely to have contributed to
the pain
B Shoulder pain causes distress but does not
. affect outcome
C A radiograph of the shoulder is not
. necessary
D Treatment with simple analgesics is a
. sensible initial approach
E. Intra-articular steroids are likely to
exacerbate the problem.
Correct Answer: D
Comment
Shoulder pain is a common complication of
stroke. The aetiology is thought to be multifactorial
and includes poor handling of the affected arm
causing trauma to the shoulder structures,
glenohumeral subluxation, abnormal tone (spasticity
and flaccidity) and pain due to the stroke itself
(central post-stroke pain). It is associated with
prolonged hospital stay and poor recovery of arm
function. A radiograph should be performed to

exclude fracture or dislocation. Treatment usually


starts with simple analgesics or non-steroidals.
Physiotherapists may treat subluxation with
strapping.If the pain persists intra-articular steroid
injections and TENS may help.

56. A 78-year-old woman found it much easier


to manage after the occupational therapist
provides several aids and appliances for use in
the kitchen. From their appearance, the
problem affecting her hands (see image) is:
A rheumatoid
. arthritis
B nodal
. osteoarthritis
C psoriatic

arthropathy

D tophaceous gout
.
E. Charcot joints.
Correct Answer: C
Comment
This is the classic appearance of arthritis mutilans
in psoriatic arthropathy, with a deforming destructive
arthritis of the small joints. Osteolysis results in
shortening of some of the digits. The pattern is not
that of rheumatoid arthritis and the wrists appear to
be spared. There is too much destruction for
osteoarthritis. The appearance of gout may be
similar, though the tophi would be easier to see in
life (as opposed to a photo). Charcot joints
(neuropathic arthropathy) usually affect the foot and
ankle.
57. A 56-year-old man presents with recurrent
attacks of polyarticular gout despite treatment
with allopurinol 300 mg daily for the last year.
He is adamant that his compliance with his
drug treatment regimen is good.
Which of the following factors is most likely to
explain his poor response to allopurinol?
A
. Concurrent treatment with
colchicine

B Fast metabolism of allopurinol


.
C High alcohol intake
.
D Poor urate clearance via the
. kidney
E. High dietary purine intake
Correct Answer: C
Comment
Persistently high alcohol consumption is a
common cause of poor response to allopurinol,
although the underlying mechanism of this is unclear.
B, D and E are plausible answers, but are less
important in practice. Most adults will respond to
allopurinol 300 mg daily, although a small proportion
will require 600 or even 900 mg daily. The aim of
treatment should be to suppress the serum urate
level to the lower end of the normal range or just
below.
58. A 68-year-old woman complained of
progressive pain in her left hip after slipping
on ice and sustaining a sub-capital fracture of
the left femoral neck hip. Following recovery
from a hemiarthroplasty, bone mineral density
(BMD) measurement of the opposite femur
confirmed the diagnosis of osteoporosis with a
T score of -2.67. She did not wish to take
oestrogen replacement therapy and was given
alendronate 70 mg weekly. Which of the
following statements is true?

A Normally the peak bone mass in a woman is


. achieved just before the menopause.
B Plain radiographs are very sensitive in
. identifying minor reduction in bone mineral
density and hence a very useful screening
test.
C The T score compares a patients BMD with
. the mean value for young, healthy adults of
the same sex.
D A high serum phosphate level may be
. suggestive of chronic alcoholism.
E. Bisphosphonates such as alendronate act by
stimulating bone formation.
Correct Answer: C
Comment
Bone mass increases during childhood and
adolescence and peaks between the ages of 20 and
30 years. Peak bone mass is influenced by age, sex,
genetic factors, hormonal status, exercise, and
calcium intake. As women in general have a lower
peak bone mass than men, it is understandable why
osteoporosis is predominantly seem in women.
Plain radiographs are not sensitive enough to
diagnose osteoporosis. Results of bone mineral
density (BMD) tests are typically reported as T
scores and Z scores.

The T score compares a patient's BMD with the


mean value for young, healthy adults of the
same sex.

The Z score compares a patient's BMD with the


mean value for persons of the same age and
sex.

Both scores are expressed in terms of standard


deviations from the mean. The World Health
Organization has defined osteoporosis as a BMD of at
least 2.5 standard deviations (SD) below the mean
value in young normal adults (i.e. T<=-2.5).
Chronic alcoholism is another important cause of low
serum phosphate level, in which case the liver
enzymes will be elevated. All bisphosphonates act
similarly on bone in binding permanently to
mineralized bone surfaces and inhibiting
osteoclastic activity. Thus, they inhibit bone
resorption and less bone is degraded during the
remodelling cycle. They do not stimulate bone
formation.
59. A 68-year-old man has lost weight. His
alkaline phosphatase is raised at 290 U/L
(normal range 35-120). Plain radiographs show
sclerotic lesions of bone.
What is the likely diagnosis?
A Stomach cancer
.
B Prostate cancer
.
C Multiple
. myeloma

D Lung cancer
.
E. Osteomalacia
Correct Answer: B
Comment
Cancer of the prostate is typically associated
with sclerotic bone lesions in contrast to the lytic
lesions seen in multiple myeloma. Osteomalacia is
associated with Looser's zones.
60. A 33-year-old woman presents with a 6month history of Raynaud's phenomenon
affecting her hands and feet. She is previously
well and takes no medication.
Which two of the following clinical features or
investigation results are the strongest
predictors that she will develop a connective
tissue disease in the future?
A Age >25 years
.
B Abnormal nail-fold capillary
. microscopy
C Elevated erythrocyte sedimentation
. rate
D History of recurrent miscarriage
.
E. Family history of Raynaud's

F. Anaemia
G Strongly positive antinuclear
. antibody
H History of chilblains
.
I. Raised platelet count
J. Dry eyes and dry mouth
Correct Answer: BG
Comment
These factors are strongly predictive of a future
connective tissue disease (CTD), particularly
abnormal nail-fold capillaries. The likelihood of
developing a CTD also increases with age of onset of
Raynaud's, with a particularly high risk in those aged
over 35 years. All the other features apart from a
family history (which suggests primary Raynaud's)
are associated with CTD, but have not been shown to
have the same predictive value as B and G.
61. A 22-year-old man has chronic diarrhoea
and has had several episodes of sinusitis and
pneumonia. A diagnosis of common variable
immunodeficiency (CVID) is made. Which two
of the following are recognized long-term
complications of this disorder?
A Type II diabetes
.
B Bronchiectasis
.

C Hyperparathyroidi
. sm
D Atheroma
.
E. Epilepsy
F. Enteropathy
G Cardiomyopathy
.
H Chronic renal
. failure
I. Optic atrophy
J. Sacroileitis.
Correct Answer: BF
Comment
CVID patients suffer from recurrent pulmonary
infections, and resultant repeated structural lung
damage can progress to bronchiectasis.
Gastrointestinal symptoms are a common problem in
CVID with a significant proportion being attributed to
a celiac like enteropathy.
62. A 40-year-old woman presents with a 6month history of a purpuric rash on her legs,
non-specific joint pains and vague ill health.
Initial investigations reveal a positive
rheumatoid factor. The GP makes a tentative
diagnosis of rheumatoid arthritis and refers
her to hospital. Results of further
investigations are as follows: Urinalysis: red

cell casts, protein+, ANA 1/80, anti-DNA


negative, anti-ENA negative, serum C3 1.02 g/l
(NR 0.75-1.65), C4 <0.02 g/l (0.20-0.65), CRP
<5mg/l (NR <5), creatinine 145 (NR 50-140).
Which is the most likely diagnosis?
A Systemic lupus erythematosus
. (SLE)
B Rheumatoid arthritis
.
C Mixed cryoglobulinaemia
.
D Primary complement deficiency
.
E. C1 inhibitor deficiency.
Correct Answer: C
Comment
This lady has a hypocomplementaemic
glomerulonephritis (note active urine sediment) and
a purpuric rash accompanied by a positive
rheumatoid factor and an isolated positive ANA.
The combination of renal and skin involvement in this
setting with a markedly low C4 suggests mixed
cryoglobulinaemia. Her serology does not support
SLE.
The key investigations which would help confirm the
diagnosis would be a warm clotted sample for
cryoglobulins and a renal biopsy.

63. A 69-year-old man with rheumatoid


arthritis presents with increased
breathlessness and nose bleeds. Which of the
following treatments for rheumatoid arthritis
are unlikely to cause bone marrow
suppression?
A Gold
.
B Azathioprine
.
C Non-steroidal anti-inflammatory drugs
. (NSAIDs)
D Methotrexate
.
E. Sulphasalazine
F. Infliximab
G Leflunomide
.
H Cyclophosphamide
.
I. Penicillamine
J. Hydroxychloroquine
Correct Answer: CF
Comment
Many of the treatments for rheumatoid arthritis
cause bone marrow suppression. NSAIDs may be
used early in the disease and cause anaemia via

gastrointestinal ulceration. Infliximab has been


associated with multiple sclerosis-like demyelination,
the development of tuberculosis and worsening heart
failure.
64. A 70-year-old female has a 17-year history
of rheumatoid arthritis. She presents with
recurrent attacks of red congested eyes with a
sensation of grittiness. The most likely cause
of her red eyes is likely to be:
A scleritis
.
B episcleritis
.
C keratitis
.
D keratoconjunctivitis
. sicca
E. choroiditis.
Correct Answer: D
Comment
Approximately 25 % of patients with rheumatoid
arthritis (RA) will have ocular manifestations.
keratoconjunctivitis sicca, scleritis, episcleritis,
keratitis, peripheral corneal ulceration, and other less
common entities such as choroiditis, retinal
vasculitis, episcleral nodules, retinal detachments,
and macular edema.

Keratoconjunctivitis sicca, or dry eye syndrome,


is the most common ocular manifestation of RA and
has a reported prevalence of 15 to 25 %. The
patient reports a gritty sensation in the eyes .
Scleritis and episcleritis as causes of recurrent red
eyes are distinguished on the basis of anatomy and
appearance. Symptoms may be similar, but the pain
in scleritis is more evident and severe. Tenderness to
palpation of the globe can help differentiate the two.
After asking the patient to look down with eyelids
closed, the physician gently presses the globe.
Patients with scleritis have tenderness on palpation,
while those with episcleritis do not.. Unlike scleritis,
patients with episcleritis do not complain of blurred
vision or photophobia. The importance of correctly
diagnosing and distinguishing between scleritis and
episcleritis is based on the potential ocular and
systemic complications associated with scleritis.
Studies have shown that patients with RA-associated
scleritis have more widespread systemic disease and
a higher mortality rate than those episcleritis.
65. A 20-year-old man with common variable
antibody deficiency presents to the Emergency
Department with a 3-day history of cough
productive of green sputum. His temperature
is 37.5C, p 84/min, respiratory rate 12/min,
and breath sounds are vesicular. Chest
radiograph is unremarkable. Which two of the
following actions do you recommend
immediately?

A Prescribe a 14 day course of antibiotics


.
B Prescribe nebulised salbutamol
.
C Give an infusion of intravenous
. immunoglobulin
D Take a sputum sample to look for acid fast
. bacilli
E. Prescribe a 5-day course of antibiotics
F. Check his serum immunoglobulin levels
G Choose an antibiotic regimen suitable for
. possible pseudomonas infection
H Order a high resolution CT scan
.
I. Take a sputum sample for culture
J. Prescribe a 28 day course of antibiotics.
Correct Answer: AI
Comment
Antibody deficient patients need prompt treatment of
presumed bacterial infection. Treatment should be
continued for slightly longer than normal; 14 days for
an uncomplicated chest infection would be
appropriate. Most infections are caused by common
organisms such as haemophilus or
pneumococcus. Psuedomonas is unusual and
mycobacterial disease rare in common variable
immunodeficiency. Cultures are invaluable if there is
a poor response to treatment and for guiding future

antibiotic choices. For infections causing fever,


routine antibody replacement should be deferred for
24-48 hours until there is a clear response to
treatment, as adverse reactions are much more
common in the presence of fever.
67. A 65-year-old woman with a mitral valve
replacement presents to the Emergency
Department with pyrexia and fainting. She is
unwell, hypotensive, anaemic and pyrexial. She
has a vague history of suffering from a
reaction to penicillin in her childhood. After
taking blood cultures she is started on broadspectrum antibiotics. Cardiac valvular
vegetations are seen on echocardiography and
her blood grows methicillin-sensitive
Staphylococci the microbiologist suggests
naficillin as the most appropriate antibiotic,
but is concerned that she may have allergy to
beta lactam-based antibiotics.
Which of the following is most appropriate to
investigate her history of possible penicillin
allergy?
A serum tryptase
.
B skin prick test to
. penicillin
C serum penicillin specific
. IgE
D patch test to penicillin

.
E. serum IgE
Correct Answer: B
Comment
A history of penicillin allergy is relatively common in
hospital patients, and is in most not due to type I
hypersensitivity reactions. A diagnosis of
penicillin allergy crucially requires a detailed history
of the drug reaction, and can be confirmed by a
positive skin prick test to the major and minor
determinants of penicillin. Skin prick testing is carried
out if there is a clinical need for penicillin treatment
e.g. treatment of infective endocarditis. A patient is
unlikely to develop anaphylaxis with a negative
penicillin skin prick test. The detection of penicillin
specific IgE in the serum is unreliable.
68. A 36-year-old man presents with a 2-day
history of severe pain in the left knee. He has
recently returned from a holiday in Spain.
Examination shows low grade fever (37.5C)
and marked synovitis of the left knee with a
tense effusion. Which are the two most likely
diagnoses?
A Lyme disease
.
B Rheumatoid arthritis
.
C Pseudogout
.

D Gout
.
E. Septic arthritis
F. Osteosarcoma
G Osteoarthritis
.
H Reactive arthritis
.
I. Torn medial meniscus
J. Spontaneous
haemarthrosis.
Correct Answer: DH
Comment
Reactive arthritis, consequent upon either genital
chlamydia trachomatis or enteric infection, is the
most likely diagnosis. He is a little younger than
many patients presenting with gout, but this is not a
rare diagnosis in this age group, particularly if
alcohol intake is high. The important diagnosis to
exclude is septic arthritis, but this is a rare cause of
monoarthritis in this age group. Aspiration of synovial
fluid is the key diagnostic tool to differentiate
between these diagnoses.
69. A 65-year-old woman with primary
Sjgren's syndrome, which was diagnosed 1
year previously, is noted to have a persistently
elevated erythrocyte sedimentation rate (ESR)

of 90-100 mm/hour and a normal C-reactive


protein (CRP). At a routine outpatient visit it is
noted that her sicca symptoms are still
troublesome, but there are no other
abnormalities on clinical examination. The
following investigations are performed:
haemoglobin 12.5 g/dL (normal range 12-16),
white cell count 6.4 x 109/L (normal range 411), platelet count 320 x 109/L (normal range
150-400), ESR 98 mm/hour, CRP <6 mg/dL
(normal <6), serum IgG 42 g/L (normal range 613), IgA 8.4 g/L (normal range 0.8-4.0) and IgM
3.6 g/L (normal range 0.4-2.0), and serum
electrophoresis shows polyclonal
hypergammaglobulinaemia.
Her persistently elevated ESR is best explained
by which of the following?
A Development of lymphoma
.
B Poorly controlled sicca symptoms
.
C Positive antinuclear antibody
.
D Positive antibodies to Ro and La
. antigens
E. Polyclonal
hypergammaglobulinaemia
Correct Answer: E
Comment

Together with fibrinogen, serum


immunoglobulins are the major driving force
causing an elevation in erythrocyte
sedimentation rate (ESR). Any disorder associated
with a persistently elevated polyclonal
hypergammaglobulinaemia, such as primary
Sjgren's syndrome, is likely to be linked with a
persistently elevated ESR. Note that her C-reactive
protein (CRP) is normal, which reflects the fact that
CRP production is not influenced by changes in
serum immunoglobulin levels.
70. A 28-year-old woman presented with
fatigue and extreme tiredness. Physical
examination revealed facial skin rash and
tenderness across the small joints of the
hands. She was concerned that she might have
systemic lupus erythematosus (SLE). Which of
the following tests when NEGATIVE will
virtually exclude the diagnosis of SLE?
A Antinuclear antibody (ANA)
.
B Anti-double stranded DNA (anti ds. DNA)
C Anti-Sm antibodies
.
D Anti-histone antibodies
.
E. Anti-Ro/SSA antibodies.
Correct Answer: A

Comment
Almost all patients with SLE have a positive
ANA test result. The ANA test is sensitive but not
specific for SLE. A negative result argues strongly
against a diagnosis of active SLE, but does not
exclude the possibility of other autoimmune
diseases.
Antibodies to Sm antigen are highly specific for
a diagnosis of SLE (>99%). However, only about
25% of patients with SLE have anti-Sm antibodies.
Anti-DNA antibodies are diagnostic of SLE
(specificity >99%). However, only 60% of patients
with SLE will have these antibodies. Therefore,
absence of anti-DNA or anti-Sm antibodies should not
exclude SLE as a diagnosis. Anti-Ro/SS-A antibodies
are found in 30% of patients with SLE. Anti-histone
antibodies are identified in small proportion of SLE
patients. They are more often seen with druginduced lupus.
71. A 60-year-old-man complained of pain in
both wrist evolving over 8 weeks. He noted
swelling around that area but denied stiffness.
On examination there was swelling and
tenderness just proximal to the wrist joints
without limitation of movement. There was
also prominent finger clubbing. Radiographs
revealed periosteal reaction over the lower end
of the radius and ulnar. Each of the following
disorders could be the cause behind this
patient complain EXCEPT?
A Mesothelioma

.
B Bronchiectesis
.
C Diabetes
. mellitus
D Crohn's disease
.
E. Whipple's
disease.
Correct Answer: C
Comment
Hypertrophic osteoarthropathy (HOA) is a
syndrome characterized by proliferative changes in
the skin and skeleton. Proliferative periostitis of the
radius and fibula and digital clubbing are commonly
seen. Two forms of the syndrome are seen, a rare
idiopathic from called pachydermoperiostitis (35%) (familial autosomal dominant), and more
common secondary form (95-97%).
Secondary HOA was initially described in
association with chronic suppurative infection and
malignancy of the lung and pleura. Therefore it used
to be called hypertrophic pulmonary
osteoarthropathy (HPOA).
Pleural causes include pleural fibroma and
mesothelioma.
Pulmonary causes include bronchogenic

carcinoma, pulmonary tuberculosis; pulmonary


abscesses, bronchiectasis emphysema; and
Pneumocystis carinii infection in patients with AIDS,
Hodgkins disease, metastases, or cystic fibrosis.
Cyanotic heart disease with a right-to-left shunt is
the only cardiac cause described.
Abdominal causes include liver cirrhosis, ulcerative
colitis, Crohn disease, Whipple disease, and biliary
atresia.
In long standing insulin dependent diabetes
mellitus, contracture of the flexor tendons of the
fingers with tight, waxy skin appearance leads to
diabetic cheirarthropathy. There is no pain and minor
functional impairment.
Diabetes mellitus is not associated with HOA.

72. A 56-year-old patient presented with 6month history of excessive tiredness, stiffness
and pain across the upper and lower limbs. On
examination he has proximal muscle weakness.
The creatinine kinase was 10 times the upper
limits of normal. The skin lesions shown in the
image are most likely to be:

A garrods pads
.
B tendon xanthoma
.
C Gottron's papules
.
D rheumatoid
. nodules
E. gouty tophi.
Correct Answer: C
Comment
Gottron's papules, violaceous papules overlying
the dorsal interphalangeal or metacarpophalangeal
areas, elbow or knee joints, occur in approximately
70% of patients with dermatomyositis.
Garrods fatty pads (knuckle pad syndrome) is a
relatively common and harmless condition
characterized by discrete fibromatous skin pads over
the dorsum of the interphalangeal joints.
73. A 36-year-old woman presents with
deteriorating nocturnal paraesthesia affecting
both hands. It improves during the morning.
Which of the following tests is least likely to be
helpful in establishing the cause?
A Thyroid function
. test

B Rheumatoid factor
.
C Tinels test
.
D MRI scan hands
.
E. Urinary hCG.
Correct Answer: D
Comment
Carpal tunnel syndrome classically presents with
nocturnal paraesthesia in the palms of the hands
which improves by shaking / using the hands. It is
caused by compression of the median nerve within
the carpal tunnel in the wrist. MRI hands would not
be indicated.
Causes of Carpal tunnel syndrome include
hypothyroidism, synovitis including rheumatoid
arthritis and pregnancy. Tinels test is a provocation
test to reproduce symptoms by percussion over the
median nerve.
74. A 47-year-old woman had a splenectomy 10
years previously. She is at increased risk of
severe infection with which of the following
organisms:
A Aspergillus nigrans
.
B Epstein-Barr virus
.

C Falciparum malaria
.
D Mycobacterium avium
. intracellular
E. Papillomavirus
F. Chlamydia trachomatis
G Mycobacterium tuberculosis
.
H Mycoplasma hominis
.
I. Pneumococcus
J. Candida albicans
Correct Answer: CI
75. An 8-year-old has been admitted for a third
time with haemolytic-uraemic syndrome and is
found to have a low C3. Which of the following
is the most likely diagnosis?
A Lupus
.
B Gram-negative
. septicaemia
C Factor H deficiency
.
D Factor I deficiency
.
E. C3 deficiency.
Correct Answer: C

Comment
Factor H deficiency accounts for 5-10% of all cases
of haemolytic-uraemic syndrome. Typically it has
a higher mortality and over half of patients will suffer
relapses. Over-activation of the alternative
complement pathway results in a low serum C3.
Although able to cause a low C3, lupus, factor I and
C3 deficiency are not associated with recurrent
haemolytic-uraemic syndrome.
76. A 50-year-old man gives an 8-month history
of episodic, painful soft tissue swellings
involving his hands, eyes and lips. There is no
temporal relationship to food. Which of the
following tests is the most useful?
A Skin prick tests to various food
. allergens
B Complement
.
C Full blood count (FBC)
.
D Glucose tolerance test
.
E. Urinary 5-hydroxyindoleacetic acid.
Correct Answer: B
Comment

The most likely diagnosis in this patient is chronic


idiopathic angioedema, However, the rarer C1-inh
deficiency has to be considered and would be
excluded by a normal C4 level.
The lack of any temporal relationship of his
symptoms to the ingestion of any food excludes an
allergic cause, which only rarely presents as chronic
angioedema.

77. A 19-year-old woman presents with fever


and cough. Sputum samples are negative on
microscopy for acid fast bacilli, but six weeks
later M. tuberculosis is grown. She completed a
course of chemotherapy for pulmonary TB two
months previously. Her chest radiograph is
unchanged from one taken at this time. Which
of the following is the most likely explanation
for these findings?
A She has HIV co-infection causing increased
. susceptibility to mycobacteria.
B The organism isolated is a contaminant.
.
C She has been re-infected with a different
. strain of TB.
D She has underlying IFNg receptor deficiency
. causing increased susceptibility to
mycobacteria.
E. She has been poorly adherent to therapy
and her TB has recurred.

Correct Answer: E
Comment
Poor adherence is a common cause of treatment
failure or early relapse; directly observed therapy
(DOT) may improve adherence if this is the case. TB
usually responds well to conventional treatment even
if the patient is co-infected with HIV. In this situation,
recurrent TB is most often caused by poor
adherence (early) or reinfection (late).
Underlying IFNg receptor/ IL12 deficiency is
extremely rare and is associated with disseminated
disease, usually with poorly pathogenic
environmental mycobacteria.. Environmental
mycobacteria may occasionally grow as
contaminants in culture, but it is most unusual for
MTB to do so.
78. The A&E SHO rings for your advice
concerning a 57-year-old man who attended
following a dog bite which required suturing.
He has had a tetanus booster and is taking
penicillin 250mg bd. Twenty four years
previously he had a splenectomy following a
road traffic accident. He has remained well
since. Which of the following statements are
true?
A Large granular lymphocytes may be visible
. on blood film.
B He should avoid killed vaccines.
.

C He is at risk of pneumocysitis pneumonia


. and appropriate long-term prophylaxis
should be given.
D Penicillin may be stopped after 10 days.
.
E. He should not have an annual flu
vaccination.
F. He is at increased risk of overwhelming
capnocytophagia canimorsus infection
following the dog bite. Appropriate
antibiotic cover should be given.
G He is at increased risk of babesiosis
. following the dog bite. Appropriate
antibiotic cover should be given.
H Howell Jolly bodies may be visible on blood
. film.
I. He is not at increased risk of overwhelming
pneumococcal infection.
J. He is at increased risk of coeliac disease.
Correct Answer: FH

79. A 30-year-old female nurse presents with a


3-month history of Raynaud's phenomenon.
Clinical examination reveals cold hands but no
other evidence of connective tissue disease.
Which of the following tests is most helpful in
determining future progression to systemic
connective tissue disease?

A Positive anti-mitochondrial antibody


.
B Positive anti-gastric parietal cell
. antibody
C Positive smooth muscle antibody
.
D Positive anti-nuclear antibody
.
E. Positive rheumatoid factor
Correct Answer: D
Comment
Although 5% of the general population have
Raynaud's phenomenon, only a minority go on to
develop systemic connective tissue disease. A
positive ANA is the single best predictor of
existing or future progression to connective
tissue disease in this situation.
80. A 38-year-old woman consults her dentist
because of dental caries. He notes that she has
a dry mouth and refers her to you with a
possible diagnosis of Sjrgrens syndrome.
Which of the following does not support this
diagnosis?
A Dry eyes
.
B Parotid

swelling

C
. Fatigue
D Arthralgia
.
E.

Pleural
effusion

Correct Answer: E
Comment
Sjrgrens syndrome results from an inflammatory
lymphocyte infiltrate of salivary glands. The most
common symptoms are dry eyes and mouth, with
fatigue. Many people do not volunteer these
symptoms unless asked, for example by the dentist,
aware that lack of saliva predisposes to caries.
Arthralgia is common in primary Sjrgrens.
Secondary Sjrgrens is associated with a variety of
connective tissue diseases, which may themselves
be a cause of arthralgia.
Tricyclic antidepressants are an unrelated cause of
dry mouth.
81. A 79-year-old man presents feeling very
unwell with recent and sudden onset of severe
pain and stiffness around his shoulders and
hips. He was previously well and admits to no
other symptoms, in particular no headache or
visual disturbance. Examination shows painful,
restricted shoulders and hips, but no

myopathy. He has no lymph nodes or


abdominal masses. Initial investigations show
a normal blood count, biochemical profile,
thyroid function, prostate specific antigen,
MSU and chest radiograph. His ESR is 85mm
and CRP is 110mg/l (normal<10). Which two of
the following statements give the best advice
regarding his treatment?
A He should be started on Prednisolone 60mg
. daily reduced over 12-18 months.
B He should be treated with a Cox II inhibitor.
.
C He should be started on Prednisolone 15mg
. daily for 2 weeks only
D He should be treated with alfacalcidol 2ug
. daily.
E. He should be treated with low dose
methotrexate.
F. He should be started on Prednislone 15mg
daily, tailed off over 12-18 months.
G He should receive 3x1g doses of i.v.
. methylprednisolone.
H He should receive prophylactic treatment for
. osteoporosis.
I. He should receive simple analgesia.
J. He should receive Prednisolone 60mg daily
for 1 month.
Correct Answer: FH
Comment

The clinical picture is highly suggestive of


polymyalgia rheumatica. This will usually respond
dramatically to small doses of prednisolone, which
will be required for an average duration of 12-18
months before the disease goes into remission. The
major side effect of these low doses of steroid is
osteoporosis, and prophylaxis will usually be
required. This decision may be informed by bone
densitometry, if available.
82. A 31-year-old man with known selective IgA
deficiency and abdominal pain is tested for
anti-endomysial antibodies (EMA). His results
are as follows: IgA endomysial antibody
negative, IgG endomysial antibody positive.
Which of the following statements is true?
A Coeliac disease can be confidently excluded
. on the basis of these results.
B His IgA deficiency is irrelevant to his
. negative IgA EMA status.
C The negative IgA EMA is of no diagnostic
. value in the presence of IgA deficiency.
D Referral for small bowel biopsy is not
. warranted on the strength of these results.
E. For diagnostic purposes, IgA and IgG EMA
are equally important
Correct Answer: C
Comment

Since there is a recognized association between


coeliac disease and IgA deficiency, it is important
to recognize that IgA EMA may be spuriously
negative in patients with coeliac disease and total
IgA deficiency, as in this case. For this reason, testing
for coeliac disease antibodies always includes
measurement of both IgA and IgG antibody isotypes
or measurement of serum IgA. Such a testing
strategy will alert both the laboratory and the
requesting clinician of 'false-negative' results as in
the case here. In keeping with the mucosal route of
antigen entry, IgA endomysial antibodies are
more important than IgG antibodies as a
diagnostic marker of coeliac disease.
83. A 75-year-old woman presents with acute
pain and swelling in the right knee. On
examination the right knee is hot and tender
with moderate synovial fluid collection.
Radiograph of the right knee shows
calcification of the meniscus. Synovial fluid was
sterile on culture and rhomboid shaped
crystals were identified on microscopy of the
fluid. Which of the following statements about
this condition is NOT true?
A Narrowing of the joint space is often present
.
B The symphysis pubis is one of many other
. joints that can be affected by this disorder
C Serum calcium levels may be elevated in
. some patients
D Negatively birefringent crystals may be

identified on synovial fluid analysis

E. There may be large subchondral cysts


Correct Answer: D
Comment
This patient presents with acute monoarthritis of the
right knee. The clinical picture and the radiological
findings are suggestive of pseudogout and calcium
pyrophosphate deposition (CPPD), which is
characterised by the presence of positively
birefringent crystals on synovial fluid analysis
(negatively birefringent ones are found in gout).
The knee is the most common site affected by
this disorder. Other locations include:

the triangular fibrocartilage of the wrist

sacroiliac joints

symphysis pubis

the glenoid of the shoulder

the labrum of the hip

the elbow

the ankle

acromioclavicular joint.
CPPD is associated with hemochromatosis,
primary hyperparathyroidism, hypothyroidism,
and hypomagnesemia. Other findings include
joint space narrowing, large subchondral cysts and
many intra-articular bodies from subchondral bone
fragmentation.

84. A 20-year-old woman presented with


sudden onset of swelling of the lips and
tongue. She also had abdominal pain and
vomiting. Her mother confirmed that her
daughter had similar attacks over the years
and even as a child. A brother and older sister
have the same disorder. Which of the following
statements about this disease is accurate?
A It has sex-linked inheritance.
.
B Animal allergen is often identified in the
. house
C Serum C4 levels are often low.
.
D Antinuclear antibodies (ANA) is often
. positive.
E. Raised IgE helps differentiate it from other
immune disorders.
Correct Answer: C
Comment
Hereditary angioneurotic oedema is an
autosomal dominantly inherited condition caused by
a deficiency of C1 esterase inhibitor. This results
in intermittent episodes of spontaneous complement
activation. Clinically the patient suffers oedema of
the skin and mucosal surfaces. Fatalities may occur if
the airway is compromised. C4 levels are typically
low during an attack; they may be normal in
between attacks.

Acquired angioedema (AAE) is angioedema


associated with allergic reactions, which is often
associated with urticaria. Approximately 94% of
cases of angioedema are drug-induced. Most are
patients taking angiotensin-converting enzyme (ACE)
inhibitors. Insect stings, and foods are other
predisposing factors.
85. A 65-year-old woman presents with a 3month history of recurrent attacks of facial
angioedema. Which of the following test
results would favour a diagnosis of C1 inhibitor
deficiency?
A Low complement C3 levels and a
. normal C4
B Low complement C4 levels and a
. normal C3
C Hypergammaglobulinaemia
.
D Hypogammaglobulinaemia
.
E. Positive rheumatoid factor
Correct Answer: B
Comment
The hallmark of all forms of C1 inhibitor
deficiency is a reduced C4 with normal C3 - a
consequence of uncontrolled complement classical
pathway activation. At this age, the most likely
diagnosis is acquired C1 inhibitor deficiency
associated with lymphoproliferative disease, or

more rarely due to autoantibodies to C1


inhibitor.
86. A 15-year-old boy presented with
arthralgia, skin rash and haematuria. Renal
biopsy showed focal necrotising
glomerulonephritis with diffuse mesangial IgA
deposits. What is the most likely diagnosis?
A Systemic lupus erythematosus
. (SLE)
B HenochScholein purpura
.
C Juvenile rheumatoid arthritis
.
D Post-streptococcal
. glomerulonephritis
E. Goodpasture's syndrome.
Correct Answer: B
Comment
HenochSchnlein purpura (HSP) is characterised
by the tissue deposition of IgA-containing immune
complexes. The pathogenesis of this disorder may be
similar to that of IgA nephropathy, which is
associated with identical histologic findings in the
kidney.
HSP occurs more often in children than in adults, and
many cases follow an upper respiratory tract
infection, suggesting that the precipitating antigen
may be infectious The clinical manifestations include

a classic tetrad that can occur in any order and at


any time over a period of several days to several
weeks: rash, arthralgias, abdominal pain, and renal
disease typically purpuric (with normal clotting
studies) and distributed symmetrically over the lower
legs and arms.
87. A 60-year-old accountant complains of
recurrent attacks of exquisite pain and
swelling in the left big toe. Which of the
following conditions is NOT likely to be
associated with this disorder?
A Chronic
. alcoholism
B Obesity
.
C Rheumatoid
. arthritis
D Diabetes mellitus
.
E. Diuretic therapy.
Correct Answer: C
Comment
Acute gout is intensely inflammatory, and is
therefore characterised by severe pain, redness,
swelling and disability. At least 80% of initial attacks
involve a single joint, typically in the lower extremity,
most often at the base of the great toe (first
metatarsophalangeal joint, known as podagra), or in

the knee. Trauma, surgery, starvation, alcohol


ingestion, dietary overindulgence, and ingestion of
drugs -diuretics (cyclosporin and low dose aspirin)
affecting serum urate concentrations may all
promote gouty attacks. Similarly chronic disorders
such as diabetes mellitus, obesity,
hyperparathyroidism and hypothyroidism are
associated with increased incidence of acute
gouty attacks. Rheumatoid arthritis is not
associated with increased incidence of
hyperuricaemia or gout.
88. A 30-year-old woman develops a systemic
reaction characterized by hypotension,
bronchospasm and widespread urticaria soon
after induction of anaesthesia for
cholecystectomy. Which of the following blood
test results would suggest that her reaction
was associated with mast cell degranulation?
A Elevated plasma
. tryptase
B Hypernatraemia
.
C Hypokalaemia
.
D Hypocomplementaemia
.
E. Hypergammaglobulinae
mia.
Correct Answer: A

Comment
The constellation of acute symptoms in this case is
highly suggestive of a systemic allergic reaction,
either anaphylaxis (if IgE mediated) or an
anaphylactoid reaction (if non-IgE mediated). Both of
these reactions are due to extensive mast cell
degranulation leading to release of large amounts of
tryptase in to the circulation. Elevated tryptase
levels in the context of this lady's reaction are
highly suggestive of an
anaphylactic/anaphylactoid reaction.
This lady will need to be investigated during
convalescence in an allergy clinic to determine the
cause of her reaction.
89. A 68-year-old woman with longstanding
congestive cardiac failure (ejection fraction
20%) presents with a hot, swollen right knee.
The following results are obtained: FBC
normal, Urea 11 mmol/l, Creatinine 196 umol/l.
Synovial fluid: many monosodium urate
crystals seen on microscopy, culture sterile.
What is the best treatment for her acute
arthritis?
A Allopurinol
.
B Colchicine 0.5mg every 2-4
. hours
C Indomethacin 50mg tds
.

D Co-codamol 30/500 every 6


. hours
E. Intra-articular corticosteroid
Correct Answer: E
Comment
This patient has acute gout, probably diureticinduced. Intra-articular corticosteroids are safe and
highly efficacious in this situation, once sepsis is
excluded.
Allopurinol has no role in the acute treatment of
gout. Colchicine at these doses is very poorly
tolerated due to GI toxicity. Non-steroidals are
very likely to precipitate a severe deterioration in
renal function and may also exacerbate heart failure.
90. A 29-year-old woman has a 3-year history
of arthralgia and Raynaud's phenomenon. A
year ago she had a miscarriage at 29 weeks,
complicated by a deep vein thrombosis.
Investigations show: Hb 11.2 Wcc 4.3 Platelets
145, ANA positive 1/160, DNA negative, ENA Ro
positive, Anticardiolipin antibodies present at
moderate titre, lupus anticoagulant present.
Which statement is true?
A She has primary antiphospholipid syndrome.
.
B She should be anticoagulated for a further 6
. months.
C She has active lupus.
.

D A high C reaction protein (CRP) and


. erythrocyte sedimentation rate (ESR) would
be consistent with active lupus.
E. Hydroxychloroquine may improve her
arthralgia.
Correct Answer: E
Comment
This history is strongly suggestive of systemic
lupus erythematosus (SLE) with secondary
antiphospholipid syndrome. Foetal loss is
common due to placental infarction and insufficiency.
She is at risk of further thrombosis and should have
lifelong anticoagulation. If she is planning further
pregnancies she will need specialist referral for
intensive monitoring.
Active lupus is associated with high ESR and
DNA antibodies, with reduced complement
(especially C4) levels. High CRP suggests the
possibility of infection.
Hydroxychloroquine is effective for joint and
cutaneous manifestations of SLE.
91. A 38-year-old man presents with 4 months
of pain, stiffness and swelling of the small
joints of his hands and feet. He has a past
history of mild psoriasis. His GP has found his
erythrocyte sedimentation rate to be elevated
at 65 mm/hour.

Which of the following clinical features would


be least likely to suggest a diagnosis of
psoriatic arthritis?
A Involvement of the distal interphalangeal
. joints
B Subcutaneous nodules
.
C Dactylitis
.
D Nail involvement
.
E. Asymmetrical arthritis
Correct Answer: B
Comment
Subcutaneous nodule formation is very strongly
suggestive of rheumatoid factor-positive rheumatoid
arthritis. All the other features occur in psoriatic
arthritis. Nail involvement is a feature of skin
psoriasis, but is also strongly associated with distal
interphalangeal joint involvement in psoriatic
arthritis.
92. A 45-year-old woman developed Raynaud's
2 years ago. She now complains of
breathlessness and skin tightness affecting her
fingers. A high-resolution CT scan shows
evidence of pulmonary fibrosis.
Which of the following tests is most likely to be
positive?

A Anticentromere antibody
.
B Anti-double-stranded DNA
. antibody
C Anti-Ro antibody
.
D
. Anti-Scl-70 antibody
E. Anti-Jo-1 antibody
Correct Answer: D
Comment
This patient has clinical symptoms suggestive of
diffuse cutaneous systemic sclerosis. Pulmonary
fibrosis and anti-Scl-70 are more common in
patients with diffuse disease. Anticentromere
antibody is associated with limited cutaneous
systemic sclerosis. Anti-double-stranded DNA
antibody is associated with systemic lupus
erythematosus. Anti-Ro antibody is associated with
lupus and primary Sjgren's syndrome. Anti-Jo-1 is
associated with polymyositis, particularly in patients
with inflammatory lung disease.
93. A 30-year-old teacher presents with a 6month history of swelling and pain involving
the distal interphalangeal joints of her hands.
The ESR is 65 mm in the first hour. What is the
most likely diagnosis?
A Generalised osteoarthritis

.
B Rheumatoid arthritis
.
C Psoriatic arthritis
.
D Systemic lupus erythematosus
. (SLE)
E. Gout.
Correct Answer: C
Comment
Arthritis with predominant involvement of the distal
interphalangeal joint occurs most often in
generalised osteoarthritis and psoriatic arthritis. The
fact that this patient is relatively young and has a
raised ESR indicates an underlying inflammatory
disease is the most likely cause of her symptoms.
Therefore psoriatic arthritis is the most likely
diagnosis in this case.
Rheumatoid arthritis and SLE are known to affect
the proximal interphalangeal (PIPs) and the
metacarpophalangeal (MCPs) joints.
Chronic gouty arthritis might involve the DIPs, but
more often it involves the MCPs and PIPs in
asymmetrical fashion with or without tophus
formation.
Examination of the skin and nail for psoriasis is very
important in confirming the diagnosis.

Hairlines in the scalp, the naval and the palms are


areas often involved in psoriasis but easily missed.
94. A 57-year-old woman is admitted to the
Emergency Department at 05.00h with
orofacial angio-oedema, having eaten a prawn
curry the previous evening. She has a past
history of hypertension and hypothyroidism,
and has recently been treated for a chest
infection. Her regular medications include
bendroflumethiazide, lisinopril and
levothyroxine. She has a history of penicillin
allergy.
On admission she is comfortable at rest and
has no urticaria. Observations are as follows:
temperature 37C, pulse 80 bpm, respiratory
rate 16/minute and BP 170/100 mmHg.
What is the most likely diagnosis?
A Angiotensin-converting enzyme inhibitor. induced angio-oedema
B Anaphylaxis due to prawns
.
C Penicillin allergy
.
D Idiopathic anaphylaxis
.
E. Diuretic allergy
Correct Answer: A
Comment

Angio-oedema may be related to use of an


angiotensin-converting enzyme inhibitor and may
occur even after apparent tolerance of the drug for
many months. In this case the drug should be
stopped or substituted.
Allergic angio-oedema is usually obvious by its fast
onset, typically within a few minutes of contact with
the allergen.

35. An 82-year-old woman is admitted with a


history of poor mobility. There is no history of
trauma and her only complaint is pain in her
left knee. Examination of the knee shows that
it is painful to touch but there are no signs of
inflammation. An abdominal radiograph

incidentally shows a left hip fracture (see


image). Why is she having knee pain?
A Pain in the knee is referred to the hip by the
. superior gluteal nerve.
B Pain in the knee is referred to the hip by the
. inferior gluteal nerve.
C Pain in the knee is referred to the hip by the
. femoral, sciatic and obturator nerves.
D Pain in the knee is referred to the hip by the
. musculocutaneous nerve.
E. Pain in the knee is an incidental finding.
Correct Answer: C
Comment
The knee pain was referred from the hip to the knee.
Branches of the femoral, sciatic and obturator
nerves all give twigs to both joints. The geniculate
branch of the obturator is the main conveyer of
pain referred from hip to knee. Referred pain
from the hip to the knee is well known to occur in
osteoarthritis of the hip but that it can occur in hip
fractures maybe less well appreciated.
Some older patients with severe osteoporosis may
sustain a hip fracture simply by turning with no
history of trauma

36. A 32-year-old woman with systemic lupus


erythematosus (SLE) and antiphosphlipid
syndrome presented with a three week history
of increasing pain in the right groin and
anterior thigh associated with excessive
fatigue and lethargy. She was limping and right
hip movement was limited with reduced flexion
and internal rotation. Although afebrile with a
temperature of 37C she was given intravenous
antibiotics and kept on her regular
maintenance dose of prednisolone at 7.5
mg/day. Magnetic resonance image (T1
weighted axial) of the hips (see image) shows
the entire femoral head is reduced in signal on
the right, indicating diffuse oedema. It also
shows subchondral collapse with flattening of
the right femoral head. A serpiginous band of
low signal surrounds an area of normal signal.
The most likely diagnosis is:
A SLE-associated acute inflammatory arthritis
. of the right hip

B Septic arthritis of the right hip


.
C Right psoas abscess
.
D Avascular necrosis of the right hip
.
E. Thrombosis of the right ilio-femoral vein.
Correct Answer: D
Comment
All of the above options are possible causes for this
patient's complaint. However, the magnetic
resonance image is almost diagnostic of
avascular necrosis (AVN). Corticosteroids such as
prednisolone are commonly used to treat diseases in
which there is inflammation, such as systemic lupus
erythematosus. Both steroid therapy and SLE could
predispose to avascular necrosis.
If septic arthritis is suspected, immediate joint
aspiration must be performed in order to obtain
cultures to determine the infectious agent.
Underlying bone changes are not typically present.
Inflammatory arthritis in general is associated with
synovial tissue thickening but again lacks significant
bone changes. Ilio-femoral vein thrombosis and
psoas abscess are not known to be associated with
structural damage of the femoral head. In
differentiating AVN from non-AVN disease of the
femoral head, MR imaging has been reported to
have a specifity of 98% and sensitivity of 97%.
Mnemonic to remember causes of AVN (taken from

Dahnert's Radiology Review Manual), "PLASTIC


RAGS":
P - Pancreatitis, Pregnancy
L - Legg-Perthes Disease, Lupus
A - Alcoholism, Atherosclerosis
S - Steroids
T - Trauma
I - Idiopathic (Spontaneous osteonecrosis of knee,
Legg-Calve-Perthes Disease, Freiberg Disease),
Infection
C - Caisson Disease*, Collagen Vascular Disease
R - Rheumatoid arthritis, Radiation tx
A - Amyloid
G - Gaucher's Disease
S - Sickle Cell Disease
*Caisson Disease: decompression sickness in divers
- ask about occupation!
Corticosteroid use and alcohol are the most
common associations in the UK.
97. A 30-year-old woman has started to
develop paraesthesia affecting the thumb,
index and middle fingers of the right hand.
This is worse in the morning and is relieved by
shaking the hand. Which of the following will
NOT exacerbate her symptoms?
A Hypothyroidism
.
B Diabetes mellitus

.
C Rheumatoid
. arthritis
D Driving a car
.
E. Frusemide
F. Oral contraceptive
pill
G Hydrocortisone
.
H Typing
.
I. Wrist held in
flexion
J. Osteoarthritis.
Correct Answer: EG
Comment
This patient has developed carpal tunnel
syndrome due to pressure on the median nerve.
Carpal tunnel syndrome (CTS) is more common in
patients with hypothyroidism, diabetes mellitus (DM),
rheumatoid arthritis (RA), osteoarthritis and those
taking the oral contraceptive pill. Mechanical factors
can also exacerbate CTS, particularly those that
increase flexion of the wrist e.g. driving and typing.
98. A 20-year-old woman is rejected as a blood
donor on account of Fe deficiency anaemia.

Which one of the following immunological


investigations is most likely to be helpful in
explaining her Fe deficiency?
A Anti-nuclear antibody
.
B Anti-neutrophil cytoplasmic antibody
.
C Anti-smooth muscle antibody
.
D Anti-glutamic acid decarboxylase
. antibody
E. Anti-endomysial antibody.
Correct Answer: E
Comment
Coeliac disease is a common cause of iron
deficiency in both general and hospital practice and
should be excluded in this patient. IgA endomysial
antibodies are highly specific markers of coeliac
disease (specificity 95-100%, sensitivity 70-90%,
positive predictive value 95-100%). IgA endomysial
antibodies are directed against tissue
transglutaminase, the recently described autoantigen
in coeliac disease.

99. A 35-year-old nurse developed Raynaud's


phenomenon six months ago (see image). The
antinuclear antibody (ANA) test was positive at
1:1000 with a speckled staining pattern. She
came to the out-patient department
complaining of excessive tiredness and
lethargy. The creatine phosphokinase (CPK)
was three times higher than the upper normal
limits, and the hand joints radiographs showed
no erosions. Which one of the following tests
would you request to establish the diagnosis?
A Anti-U1 RNP
.
B Anti-ds DNA
.
C Anti-Scl 70
.
D Anti-centromere

antibodies

E. Anti-PM-Scl
Correct Answer: A
Comment
The picture shows puffy hands, and fingers with swan
neck and boutonniere deformities. The fact that she
had gross deformities of the hands without joint
damage or erosions, might suggest that she has
Jaccoud's arthropathy, which is more often seen in
patients with connective tissue disease such as
systemic lupus erythematosus (SLE). This patients
also exhibits features probably suggestive of an
overlap between SLE, myositis and scleroderma
which constitute the diagnosis of mixed connective
tissue disease (MCTD).
MCTD is an overlap syndrome characterised by
combinations of clinical features of SLE, systemic
scleroderma and polymyositis. The presenting
symptoms of MCTD are most often:

Raynaud's phenomenon

puffy hands

arthralgias

myalgias

fatigue.
The various features of the connective tissue
disorders making up MCTD develop over months and
years. A defining feature of MCTD is the presence
of antibodies against the U1 ribonucleoprotein
(U1 RNP) complex, and hence the presence of high
titre anti-U1 RNP will confirm the clinical diagnosis of

MCTD. Although the other tests might also be


positive, they will not be helpful in establishing the
underlying cause of the whole clinical spectrum.
100. A 64-year-old man presents to A&E with a
2-day history of increasing pain and swelling of
his left knee. He denies a history of trauma. On
examination, the knee is hot, red, swollen and
extremely tender. Which of the following
investigations is most important?
A Plain radiograph of the
. knee
B Blood cultures
.
C C-reactive protein (CRP)
.
D Joint aspiration
.
E. Plasma uric acid level.
Correct Answer: D
Comment
Aspiration of the knee with microscopy to look for
pus cells in the case of septic arthritis and crystals in
the case of gout or pseudogout is the most useful
investigation. Blood cultures may yield an organism.
The radiographs are really as a baseline and are
unlikely to show acute changes. Uric acid levels
may not be elevated in an acute episode of
gout. A raised CRP is non-specific, but can be used
to monitor the effectiveness of treatment.

101. The parents of a 10-year-old asthmatic


boy with peanut allergy are concerned about
the risk of future anaphylaxis if he were to
inadvertently ingest peanuts. Which of the
following features is the single most important
predictor of anaphylaxis in this situation?
A Level of peanut-specific IgE in his serum
.
B Strength of positive skin test response to
. peanut
C Poorly controlled asthma
.
D Previous steroid therapy
.
E. Family history of nut allergy.
Correct Answer: C
Comment
Poorly controlled asthma is an important risk
factor for fatal anaphylaxis in this situation. Children
such as this should have their asthma well controlled
and in addition, have ready access to self-injectable
adrenaline.
102. A 55-year-old woman presents with a 3month history of a painful swollen knee.
Examination shows restricted, painful
movement and a moderate sized effusion.
Synovial fluid is aspirated for diagnostic
purposes. Which two of the following findings

in the synovial fluid would be most compatible


with a final diagnosis of osteoarthritis?
A Macroscopically clear fluid
.
B Needle shaped crystals on microspcopy
.
C Highly viscous fluid
.
D High numbers of neutrophils on
. microscopy
E. Blood stained fluid
F. Thin, watery fluid
G High numbers of macrophages on
. microscopy
H Rhomboid-shaped crystals on
. microscopy
I. Macroscopically turbid
J. Gram positive cocci on gram stain.
Correct Answer: AC
Comment
Fluid aspirated from an osteoarthritic joint is like
normal synovial fluid: typically clear, viscous and
virtually acellular. Inflammatory fluid is turbid
(reflecting a high cell count) and loses viscosity
because of release of matrix degrading enzymes in
the synovial fluid.

103. A 32-year old woman with systemic lupus


erythematous (SLE) is seen in clinic. She had a
flare of her disease 3 months ago, with
arthralgia and rash, but has been
asymptomatic since a short course of moderate
dose prednisolone. She currently takes
hydroxychloroquine 200 mg bd and
prednisolone 8mg od. Clinical examination is
unremarkable. Her ESR is 60 mm/h and her CRP
is 8 mg/dl. Which two of the following
investigations could explain her discordant
inflammatory markers?
A Serum immunoglobulins
.
B Chest radiograph
.
C Plasma viscosity
.
D Urinalysis
.
E. Complement levels
F. Plasma Lipids
G Plasma urea
.
H Serum ANA
.
I. Serum Anti-ds DNA
antibodies
J. Serum ANCA.

Correct Answer: AF
Comment
The C-reactive protein is a more sensitive marker
of inflammation and infection than the ESR. The
normal CRP in this lady fits with the clinical picture of
disease inactivity. Therefore, the raised ESR must
be secondary to a cause other than inflammation or
infection. Such causes include hyperlipidaemia,
anaemia and hypergammaglobulinaemia, all of
which frequently occur in SLE.
104. A 32-year-old man presents with a 4month history of back pain. The pain is worse
in the morning and after sitting watching TV.
Plain radiograph of the spine/pelvis shows
evidence of sacroiliitis. Each of the following
clinical features might be identified on clinical
examination of this patient EXCEPT?
A Keratoderma
. blenorhegica
B Rheumatoid nodule
.
C Onycholysis
.
D Uveitis
.
E. Urethritis.
Correct Answer: B

Comment
The long (> 1 hr) early morning and rest stiffness are
highly suggestive of an underlying inflammatory
condition. The pelvic radiograph confirms the
presence of sacroiliitis. Bilateral sacroiliitis are typical
for ankylosing spondylitis. The frequency of
asymmetric sacroiliitis may be higher in other
spondyloarthropathies, e.g., reactive arthritis,
Reiters syndrome, spondylitis associated with
psoriasis, or inflammatory bowel disease.
Reiter's syndrome is characterised by a triad of
arthritis, urethritis, and conjunctivitis. Reiter's
syndrome develops in the setting of postdysenteric
or postvenereal illness. The characteristic rashes of
keratoderma blennorrhagicum and circinate
balanitis may be present.
Psoriatic spondyloarthropathy is characterised by
psoriatic plaques. The skin involvement may be
subtle and should be searched for carefully. The cleft
of the buttock, scalp hairline, and penis are site often
involved but may be missed easily if thorough
examination of these areas was not actively
conducted. Psoriatic nail changes include
onycholysis, yellow nails and nail pitting.
Inflammatory bowel disease consists of ulcerative
colitis and Crohn's disease.
A diagnosis of ankylosing spondylitis may be made
when specific features of Reiter's syndrome,
psoriasis, or inflammatory bowel disease are absent.

Sacroiliitis is also encountered in tuberculosis,


sarcoidosis and brucellosis.
Rheumatoid arthritis is not associated with lumber or
sacroiliac joint disease. However cervical spondylitis
and atlanto-axial subluxation is not an uncommon
feature of RA

105. A 58-year-old woman is referred with pain


and stiffness in her hands and knees. She has a
few patches of psoriasis on her arms. Her
hands are shown in the picture. What are the
two most likely diagnoses?
A Gout
.
B Nodal osteoarthritis
.
C Pseudogout
.

D Systemic sclerosis
.
E. Psoriatic arthritis
F. Systemic lupus
erythematosus
G Ankylosing spondylitis
.
H Rheumatoid arthritis
.
I. Reactive arthritis
J. SAPHO syndrome.
Correct Answer: BE
Comment
Swelling of the distal interphalangeal joints really
only occurs in nodal osteoarthritis and one of the
forms of psoriatic arthritis, which is usually easily
distinguished by nail involvement (not present in this
case).
Involvement of the base of the thumb is also
pathognomonic of nodal osteoarthritis, giving the
thumb base a characteristically square appearance.

Das könnte Ihnen auch gefallen